SlideShare ist ein Scribd-Unternehmen logo
1 von 22
Downloaden Sie, um offline zu lesen
EXAMEN NACIONAL DE MEDICINA 2021
wwwww.qxmedic.com
www.grupoqxmedic.com
1. Se evidencia el incremento de pacientes
afebriles que acuden a la consulta y el servicio
de salud ambiental informa la presencia de
Aedes aegypti en el ámbito territorial del
centro de salud. En este caso, ¿cuál será la
acción prioritaria de los equipos extramurales
de salud?
A. Notificación internacional inmediata
B. Control de contactos y fuentes de infección
C. Aplicación de vacuna contra el dengue
D. Tratamiento antibiótico domiciliario de casos
2. Lactante de 9 meses presenta hace 4 días
fiebre, deposiciones con moco y sangre, pujo y
tenesmo. Al examen: signos de deshidratación
moderada, somnoliento, abdomen distendido,
doloroso, RHA aumentados. Se envía muestra
para coprocultivo. Se inicia hidratación
parenteral y se administra...........
A. Amoxicilina
B. Cloranfenicol
C. Ceftriazona
D. Ampicilina
3. Lactante varón de 6 meses, alimentado con
sustituto de leche materna. Inicia hace dos días
con fiebre, rinorrea y tos. Al tercer día se añade
dificultad respiratoria por lo que es traído a
consulta. Niega antecedentes patológicos. Al
examen: taquipneico con politirajes y
sibilancias espiratorias. ¿Cuál es el diagnóstico
probable?
A. Resfrío común
B. Cuerpo extraño
C. Bronquitis
D. Bronquiolitis
4. Varón de 33 años, acude al servicio de urología
por presentar disminución de volumen
testicular. Antecedente: hace un año sometido
a cirugía por traumatismo a nivel de pelvis.
¿Qué complicación quirúrgica explica este
cuadro?
A. Ablación del cuerpo del epidídimo
B. Ligadura de arteria testicular
C. Sección del conducto deferente
D. Desgarro de la arteria funicular
5. Paciente de 45 años, procedente de distrito
endémico del dengue, acude al puesto de salud
por presentar fiebre persistente, dolor retro
ocular, dolor abdominal intenso y vómitos.
¿Cuál es la clasificación inicial del caso?
A. Brote epidémico por dengue
B. Sospechoso de dengue clásico
C. Sindrome de choque por dengue
D. Probable dengue con signos de alarma
6. Mujer de 21 años, con perfil lipídico: colesterol
total 390 mg/dl, triglicéridos 125 mg/dl, HDL 45
mg/dl y LDL 320 mg/dl. Al examen: xantomas
en talón de Aquiles y extensores de las manos.
Padre con colesterol y LDL muy aumentados.
Hermana con colesterol de 400 mg/dl. Madre y
hermano sin alteraciones. ¿Cuál es el trastorno
más probable?
A. Hipercolesterolemia poligénica
B. Hiperlipidemia familiar combinada
C. Hipercolesterolemia familiar
D. Hipertrigliceridemia familiar
7. Mujer de 68 años, consulta por sensación bulto
y sequedad en zona vaginal. Al examen: se
evidencia distopia genital grado I según POPQ.
¿Cuál es el manejo más adecuado?
A. Estrógenos locales
B. Colpoperineoplastía
C. Uso de pesario
D. Observación y control en 1 año
8. Mujer de 22 años, consulta por flujo vaginal
amarillento, espumoso, con mal olor y prurito
vaginal. Al examen: se encuentra colpitis en
fresa, ¿cuál es el agente causal?
A. Gardnerella vaginalis
B. Trichomonas vaginalis
C. Herpes simple
D. Candida albicans
9. Segundigesta de 12 semanas, asintomática,
con sobrepeso y lupus eritematoso sistémico
no activo. Antecedente: preeclampsia con
criterios de severidad y parto pretérmino en
última gestación. ¿Qué medida sería la más
efectiva como profilaxis para la preeclampsia?
A. Dieta baja en sal
B. Ácido acetil salicílico a dosis bajas
C. Suplemento de vitamina D
D. Enoxaparina
EXAMEN NACIONAL DE MEDICINA 2021
wwwww.qxmedic.com
www.grupoqxmedic.com
10. Varón de 28 años, llevado a emergencia por los
bomberos, quienes refieren haberlo
encontrado "tirado en la calle". Al examen: FC
62 x', FR 12 x', SaO2 90%, T 35.6 °C, PA 100/60
mmHg, soporoso, pupilas fotoreactivas. Se le
administra flumazenilo 0.2 mg EV y el paciente
despierta. ¿Qué sustancia química ingirió el
paciente?
A. Alcohol
B. Antidepresivos
C. Morfina
D. Benzodiacepinas
11. Varón de 45 años sufre accidente de tránsito.
Presenta dolor, intensidad 9/10 e impotencia
funcional. Se diagnostica luxación posterior de
cadera, la reducción es realizada a las 72 horas.
¿Cuál es complicación ósea que se podría
presentar a mediano o largo plazo?
A. Anquilosis ósea
B. Necrosis avascular
C. Fractura del acetábulo
D. Lesión del nervio ciático
12. Niño de 3 años, hace 7 días inicia con edema en
párpados y tobillos. Laboratorio: colesterol 280
mg/dl, albúmina 1.5 mg/dl, proteinuria 40
mg/m2/hora, creatinina sérica 1.1 mg/dl,
hematíes en orina 3 a 4 x campo, PA sistólica
percentil 50, C3 normal. ¿Cuál es el diagnóstico
probable?
A. Sindrome nefrótico
B. Insuficiencia renal
C. Sindrome nefrítico
D. Glomerulonefritis
13. Paciente de 23 años, con diagnóstico de asma
bronquial desde su niñez, viene por crisis que
se inició hace 3 horas. Al examen PA 110/70
mmHg, FC 130 x', FR 32 x', despierto con
politirajes y silencio en la auscultación ambos
hemitórax. De todo lo expuesto, ¿qué indica
una peor evolución?
A. Politirajes
B. Polipnea
C. Silencio auscultatorio
D. Duración de la crisis
14. Varón de 60 años, consulta por aparición de
tumoraciones cervicales y axilares indoloras. Al
examen: PA 110/70 mmHg, FC 89 x', T 38.9 °C,
se palpan adenopatías cervicales y axilares
indoloras, de consistencia elástica. Resto sin
alteraciones. ¿Cuál es el procedimiento más
recomendable para el diagnóstico?
A. Extirpación de ganglio cervical y estudio
anatomopatológico
B. Tomografía de tórax, abdomen y pelvis.
C. Biopsia de médula ósea por aspiración
D. Endoscopía digestiva y fibrobroncoscopía
pulmonar
15. Varón de 44 años, se le realiza endoscopía
digestiva alta, evidenciándose zona de
excoriación de mucosa a nivel de curvatura
menor, cerca del antro y del píloro. ¿Cuál es la
causa básica que origina este hallazgo?
A. Aumento de producción de moco por las
glándulas gástricas pilóricas profundas
B. Desequilibrio entre la secreción gástrica y el
grado de protección de la mucosa
C. Deficiente acción neutralizante biliar frente al
ácido gástrico
D. Aumento de la secreción de bicarbonato de
sodio
16. Se desea estudiar el impacto de elaborar
preguntas de opción múltiple con 3
alternativas de respuesta versus 5 alternativas.
¿Qué tipo de diseño de investigación debe
aplicarse?
A. Estudio experimental
B. Casos y controles
C. Estudio descriptivo
D. Estudio de cohortes
17. Paciente de 66 años admitido en UCI por
trauma craneoencefálico grave que progresa a
diagnóstico establecido de muerte cerebral. En
junta médica se decide suspender el soporte
ventilatorio. ¿Qué valor ético se estaría
vulnerando?
A. Solidaridad
B. Respeto a la persona
C. Ninguno
D. Justicia
EXAMEN NACIONAL DE MEDICINA 2021
wwwww.qxmedic.com
www.grupoqxmedic.com
18. En una zona urbana de extrema pobreza hay
una alta concentración de casos positivos de
tuberculosis pulmonar. El centro de salud
informa que tiene asegurados los
medicamentos para el tratamiento completo
de los pacientes, sin embargo, algunos
pacientes presentan serias limitaciones para
cumplir el tratamiento supervisado. ¿Cuál es la
principal razón para esta falta de adherencia?
A. Insuficiente conocimiento sobre la enfermedad
B. Horario laboral
C. Ubicación de la vivienda
D. Disponibilidad de personal
19. Escolar de 6 años, presenta malestar general,
anorexia, dolor abdominal periumbilical de 12
horas de evolución, acompañado de vómitos
sin contenido alimentario, no diarreas. Al
examen: dolor a la palpación en fosa iliaca
derecha, ¿cuál es el diagnóstico probable?
A. Apendicitis aguda
B. Invaginación intestinal
C. Gastroenterocolitis
D. Diverticulitis
20. En su SERUMS usted evalúa a una mujer de 19
años que suele acudir con su pareja por
presentar dolor pélvico crónico e infecciones
pélvicas a repetición. Siempre abandona
tratamiento. Al examen: se muestra ansiosa,
incómoda, evita contacto visual y mira
constantemente a su pareja. ¿Cuál sería su
principal sospecha diagnóstica?
A. Sindrome depresivo
B. Endometriosis moderada
C. Violencia doméstica
D. Sindrome conversivo
21. Lactante de 8 meses, traído a consulta por
presentar desde hace 5 días fiebre, rinorrea y
tos esporádica. Hoy el niño presenta llanto
constante, rechaza la alimentación y se golpea
compulsivamente con ambas manos las orejas.
¿Qué examen confirmaría el diagnóstico?
A. Otoscopía
B. Visualización de la faringe
C. Auscultación de los pulmones
D. Búsqueda de reflejo rojo
22. En una análisis de heces de rutina se
encuentran huevos de Strongyloides
stercoralis. ¿Cuál de las siguientes opciones
sería el tratamiento de primera elección?
A. Centro de Salud Mental Comunitario
B. Mebendazol
C. Ivermectina
D. Albendazol
23. Puérpera inmediata con diabetes gestacional.
Usted desea valorar en el posparto su estatus
metabólico para manejo. ¿En qué momento le
indica dosaje de glucosa?
A. A la semana
B. De 1 a 3 días
C. A las cuatro semanas
D. A los 15 días
24. Mujer de 47 años, obesa e hipertensa. Acude a
consultorio de ginecología por presentar
sangrado uterino luego de 4 años de
amenorrea. Cérvix uterino normal. PAP
negativo. Ecografía transvaginal: endometrio
6mm. ¿Cuál es la prescripción adecuada?
A. Histerectomía
B. Biopsia de endometrio
C. Observación
D. Tomografía contrastada abdominopélvica
25. Varón de 22 años, trasladado por el SAMU por
sufrir asalto con arma blanca. Al examen:
herida profunda sangrante en la nuca. No
puede movilizar el hemicuerpo derecho y la
sensibilidad está afectada en el lado
contralateral. ¿Qué estudio es recomendable
para el diagnóstico?
A. Ecografía de partes blandas
B. Gammagrafía cervical
C. TAC cervical contrastada
D. Resonancia magnética cervical
EXAMEN NACIONAL DE MEDICINA 2021
wwwww.qxmedic.com
www.grupoqxmedic.com
26. Primigesta de 32 semanas por FUR, acude a
Emergencia por dolor abdominal tipo cólico y
pérdida de moco. Movimientos fetales
normales. Al examen: contracciones uterinas
de 30 segundos de duración cada 4 minutos,
intensidad ++. Latidos fetales 158 x'. No
pérdida de líquido a la maniobra de Valsalva.
Tacto vaginal: cérvix blando, dilatación 1 cm,
membranas íntegras. ¿Cuál es la conducta a
seguir?
A. Hospitalización e inducción del parto
B. Reposo en casa
C. Maduración pulmonar y tocólisis
D. Término de gestación por cesárea
27. Primigesta de 40 semanas en trabajo de parto.
Luego de 2 horas no presenta modificaciones
en la dilatación cervical. Dinámica uterina 2/10,
intensidad ++/+++, duración 40 segundos.
Tacto vaginal: dilatación de 6 cm, altura de
presentación -3, membranas ovulares rotas
con líquido claro. ¿Cuál es la conducta a seguir?
A. Colocar a la paciente en decúbito lateral
izquierdo
B. Cesárea
C. Acentuación del trabajo de parto
D. Reevaluación clínica de la pelvis y del feto
28. Niño de 8 meses que presenta vómitos y
diarrea, se diagnostica deshidratación
moderada y se decide dar hidratación oral. El
peso actual es de 8 kg. Se prescribe sales de
hidratación oral a razón de por 4 horas
A. A voluntad cada hora
B. 200 ml cada hora
C. 100 ml cada hora
D. 50 ml cada hora
29. Varón de 68 años con EPOC, acude a
Emergencia por presentar súbitamente dolor
torácico y disnea que no responde a
oxigenoterapia ni a broncodilatadores. Al
examen: 110/60 mmHg, FC 110 x', FR 30 x',
SaO2 86%, hiperresonancia a la percusión de
hemitórax derecho. ¿Cuál es el diagnóstico
probable?
A. Neumotórax agudo
B. Embolismo pulmonar
C. Exacerbación del EPOC
D. Neumonía lobar
30. Lactante de 1 año con rechazo al alimento
desde hace 1 mes. Al examen: desnutrido,
pálido, cefalohetoma parietal izquierdo,
piernas con lesiones hipocrómicas curvadas de
0.8 y 10 mm de longitud, manchas equimóticas
redondeadas en muslos y zona escrotal. ¿Cuál
es el diagnóstico probable?
A. Sepsis fulminante
B. Maltrato infantil
C. Enfermedad de Kawasaki
D. Déficit de antitrombina
31. Paciente usuaria de DIU acude por retraso
menstrual de tres días. Se le solicita un test de
embarazo en orina y sale positivo. Al examen:
se aprecian los hilos del dispositivo. ¿Cuál es la
conducta a seguir?
A. Repetir test de embarazo
B. Solicitar una ecografía transvaginal
C. Solicitar un β-hCG cuantitativa seriada
D. Retirar el dispositivo intrauterino
32. Recién nacido de 3500 g con hipotermia, débil
succión, hipoactividad, signos de dificultad
respiratoria, polipneico y tiraje subcostal.
Leucocitos 3000 x mm3. Relación
abastonados/neutrófilos de 0.3. ¿Cuál es el
tratamiento de primera elección luego de la
toma de cultivos?
A. Ceftriaxona + amikacina
B. Ampicilina + amikacina
C. Vancomicina + cefotaxima
D. Ampicilina + cefotaxima
33. Varón de 50 años, fumador pesado, desde hace
1 año viene presentando episodios de dolor
opresivo retroesternal que duran 5 minutos
aproximadamente, desencadenados luego de
practicar deportes y que mejora con
nitroglicerina. ¿Cuál es el diagnóstico más
probable?
A. Angina de esfuerzo
B. Infarto agudo de miocardio
C. Estenosis mitral
D. Angina estable
EXAMEN NACIONAL DE MEDICINA 2021
wwwww.qxmedic.com
www.grupoqxmedic.com
34. Taxista de 42 años, desde hace 1 año lesiones
en piel y desde 6 meses hinchazón y dolor
articular. Al examen: lesiones cutáneas,
eritematosas descamativas y pruriginosas en
codos. Laboratorio: ANA, factor reumatoide y
serología de lúes negativos; VSG 50 mm/h. RX
mano: erosiones periarticulares en
interfalángicas distales de 2do y 3er dedo de
mano derecha. ¿Cuál es el diagnóstico
probable?
A. Artritis reumatoide
B. Artritis psoriásica
C. Síndrome de Reiter
D. Artropatía amiloide
35. Mujer de 50 años que presenta sensación
persistente de tristeza y desesperanza,
dificultad para concentrarse e ideas suicidas.
Refiere que últimamente escucha voces y ve
personas que sus familiares ni oyen ni ven.
¿Cuál es el diagnóstico más probable?
A. Trastorno afectivo estacional
B. Depresión mayor
C. Distimia
D. Depresión psicótica
36. La enfermera comunica al médico jefe que va a
devolver las vacunas para el Covid-19 porque la
población no solicita su aplicación. El médico
encuentra que el 90% de la población ha
recibido la primera y segunda dosis, y solo el
30% la tercera dosis. Convoca a todo el
personal para formar brigadas de vacunación a
domicilio. La campaña resulta exitosa y se logra
la cobertura. ¿Qué habilidad de gestión ha
aplicado en este nivel el médico jefe?
A. Comunicación
B. Profesionalismo
C. Adaptación al cambio
D. Liderazgo
37. Al servicio de urgencia del centro de salud
acude un número de personas por mordedura
de perro. Refieren que son perros callejeros y
no es posible su captura, ante eso el médico
jefe decide realizar una coordinación
intersectorial. ¿A qué institución deberá
convocar prioritariamente?
A. Municipalidad
B. Fiscalía de prevención
C. Policía Nacional
D. Dirección de educación
38. Gestante de 38 semanas con IMC 24, se
muestra preocupada porque en todo el
embarazo ha ganado 15 kg. Según la mayoría
de los autores, la ganancia ponderal para ella
es:
A. Muy alta
B. Alta
C. Baja
D. Adecuada
39. Varón de 65 años, diabético desde hace 20
años, ingresa por emergencia con trastorno de
conciencia y respiración de Kussmaul.
Laboratorio: glucosa 400 mg/dl, pH 7.23, pCO2
30 mmHg, HCO3 18 mEq/L. Orina: presencia de
betahidroxibutirato. ¿Qué cambios fisiológicos
esperaría encontrar?
A. Disminución de la excreción renal de
bicarbonato
B. Disminución de la excreción de amonio en la
orina
C. Aumento de la excreción pulmonar de amonio
D. Disminución de la eliminación pulmonar de
CO2
40. Varón de 39 años con diagnóstico COVID-19,
acude a urgencia con su hijo de 13 años quien
presenta fiebre, cefalea y dolor de garganta. El
padre refiere que hace dos semanas en su
centro laboral se diagnosticaron varios casos
de COVID-19. ¿A qué tipo de epidemia
corresponde?
A. Problemas de conectividad en la educación
virtual
B. Exposición continuada
C. Fuente común
D. Mixta
41. Un médico prescribe antibióticos de marca a un
niño "porque los genéricos no son tan
efectivos". La madre del paciente hará un
esfuerzo para adquirirlos a pesar de que carece
de recursos económicos. ¿Qué principio
bioético se ha vulnerado?
A. Justicia
B. Autonomía
C. Beneficencia
D. Maleficencia
EXAMEN NACIONAL DE MEDICINA 2021
wwwww.qxmedic.com
www.grupoqxmedic.com
42. Madre refiere que su hijo de 2 años presenta
hace 24 horas dolor abdominal tipo cólico, se
acompaña de náuseas y vómitos. Abdomen: se
palpa masa alargada dolorosa en epigastrio,
RHA aumentados con timbre metálico. RX
abdomen simple: niveles hidroaéreos de asas
intestinales delgadas. TAC de abdomen: signo
de la "salchicha", líquido libre en cavidad
aproximadamente 200 cm3. ¿Cuál es la
conducta terapéutica inmediata a seguir?
A. Enema con aire
B. Colonoscopia
C. Colon con enema
D. Laparotomía exploratoria
43. Mujer de 30 años, luego un expulsivo
prolongado da a luz a un bebé de 4100 g,
alumbramiento normal. En el puerperio
inmediato presenta sangrado vaginal
abundante. Al examen: PA 50/20 mmHg, FC
120 x', sopor y palidez marcada. Al nivel de la
cicatriz umbilical se pala útero blando. ¿Cuál es
el diagnóstico correcto?
A. Laceración cervical
B. Tromboastenia de Glanzmann
C. Acretismo placentario
D. Atonía uterina
44. Niño de 2 años que, en ambiente poco
iluminados, se choca con las cosas, siente
molestia con la luz. En la evaluación presenta
xerosis conjuntival y manchas de Bitot. ¿Qué
deficiencia vitamínica se relaciona con estos
hallazgos?
A. K
B. C
C. E
D. A
45. Mujer de 75 años con hipertensión arterial y
anticoagulación por fibrilación auricular.
Llevada a Emergencia por trastorno de
conciencia. Al examen: PA 208/108 mmHg, FC
108 x', FR 16 x', SaO2 96%, T 36.8 °C, afasia y
hemiparesia derecha. Laboratorio: INR 1,6.
¿Cuál es la primera acción a realizar?
A. Reducir la presión arterial sistólica < 185 mmHg
B. Trombólisis para revascularizar
C. Optimizar anticoagulación
D. Administrar vitamina K
46. Varón de 40 años es traído por familiares a
Emergencia e ingresa a Trauma Shock, por
dificultad respiratoria progresiva. Usted
observa que el paciente deja de respirar, no
responde al llamado y no hay pulso carotídeo.
¿Cuál es el primer paso de actuación?
A. Interconsulta a cardiología
B. Desfibrilación inmediata
C. Intubación orotraqueal
D. Compresión cardiaca
47. Madre con su hijo de 5 meses acude a control
de crecimiento y desarrollo, solicita
orientación para inicio de alimentación
complementaria. Si se requiere prevenir la
anemia por deficiencia de hierro. ¿Qué
indicación se le dará?
A. Priorizar papillas en base a frutas y sopas
B. Alimentar con sucedáneos de leche materna
C. Incluir diariamente alimentos de origen animal
D. Dar término a la lactancia materna
48. Mujer de 45 años luego de larga caminata
presenta dolor con aumento de volumen de la
rodilla izquierda. Artrocentesis: leucocitosis de
12000 x mm3. ¿Cómo clasificaría a este
líquido?
A. Séptico
B. Normal
C. Inflamatorio
D. Hemorrágico
49. Varón de 23 años sufre atricción por caída de
pared sobre ambos miembros inferiores, luego
de unas horas es llevado de emergencia por
presentar oligoanuria. ¿Qué examen requiere
para establecer el diagnóstico?
A. Hematocrito seriado
B. Sedimento urinario
C. CPK total
D. Sodio sérico
50. Mujer de 34 años, consulta por amenorrea de
8 meses, pérdida de cabello y sequedad
vaginal. IMC 24. Laboratorio: FSH 44 mUI/ml,
estradiol 20 pg/ml, prolactina 10 ug/dl. ¿Cuál
es el diagnóstico más probable?
A. Falla ovárica prematura
B. Síndrome de ovario poliquístico
C. Hiperprolactinemia
D. Síndrome de Cushing
EXAMEN NACIONAL DE MEDICINA 2021
wwwww.qxmedic.com
www.grupoqxmedic.com
51. Mujer de 74 años refiere pérdida brusca de la
visión del ojo derecho con dolor intenso.
Antecedente de asma bronquial y de alergia a
las sulfonamidas. La lámpara de hendidura
revela edema del epitelio corneal, ángulo
esclerocorneal cerrado y cámara anterior poco
profunda. ¿Cuál es el tratamiento a indicar?
A. Manitol
B. Dorzolamida
C. Timolol
D. Latanoprost
52. Gestante de 28 semanas con pielonefritis
aguda en resolución. ¿Cuál es el manejo para
prevenir la reinfección en esta gestación?
A. Amoxicilina 500 mg diario
B. Urocultivo semanal
C. Examen completo de orina semanal
D. Nitrofurantoina 100 mg diario
53. Al inicio de la pandemia COVID-19 no se
contaba con vacuna, por lo cual, para disminuir
el riesgo de contagio, solo se tenía evidencia de
protección mediante el uso de mascarilla,
lavado de manos, distanciamiento físico. ¿A
través de qué intervención se logró que la
población cumpla con estas medidas?
A. Uso de medio masivos de comunicación
B. Educación a domicilio
C. Escenificación teatral en parques
D. Charlas en centros laborales
54. Varón de 68 años, en terapia con
hidroclorotiazida. Desde hace 15 días presenta
debilidad muscular, incluso se ha caído en dos
ocasiones. Al examen: presenta debilidad
generalizada con hiporreflexia. ¿Cuál es el
procedimiento inicial?
A. TAC cerebral
B. Electromiografía
C. Electroencefalografía
D. Electrocardiografía
55. Un tamizaje de violencia familiar realizado por
profesionales del consultorio de salud de la
mujer reveló que la incidencia es alta,
decidiéndose que las víctimas recibieran
atención integral especializada refiriéndolas a:
A. Centro de Salud Mental Comunitario
B. Ministerio de la Mujer
C. Hospital de tercer nivel
D. Instituto Nacional de Salud Mental
56. Mujer de 62 años con antecedente de
hipertensión arterial. Presenta bruscamente
hemiplejia derecha con afasia no afluente que
revierte íntegramente luego de 15 minutos.
Tomografía cerebral normal. ¿Cuál es el
diagnóstico?
A. Hemorragia subaracnoidea
B. Hemorragia intraparenquimal
C. Embolia cerebral
D. Ataque isquémico transitorio
57. Varón de 28 años ingresa a Emergencia por
atropello vehicular. Al examen: PA 100/60
mmHg, FC 110 x', FR 28 x', SaO2 92% con una
FiO2 40%, herida contuso cortante en región
parietal derecha, abdomen distendido, RHA
presentes y fractura fémur derecho. Al
estímulo doloroso abre los ojos, emite sonidos
incomprensibles y vuelve a caer en sopor.
¿Cuál es la primera acción a seguir?
A. Intubación orotraqueal
B. Suturar herida de la cabeza
C. Colocar vía central
D. Radiografía de cráneo
58. Mujer de 50 años consulta por dolor e
hinchazón de pierna izquierda de una semana
de evolución. Al examen: Aumento de volumen
de pierna izquierda. No signos inflamatorios.
Signo de Homans positivo. Dímero D: negativo.
¿Cuál es el diagnóstico más probable?
A. Trombosis venosa profunda
B. Paniculitis septal
C. Infección de partes blandas
D. Linfangitis crónica
59. Varón de 62 años fumador crónico de 2
paquetes diarios de cigarrillos. Desde hace 3
meses, tos persistente con expectoración
hemoptoica y baja de peso 5 kg. BK negativo
por 3 veces. RX de tórax: masa suprahiliar
izquierda de 5 cm de diámetro. ¿Qué examen
confirmaría el diagnóstico?
A. Broncofibroscopía con biopsia
B. TAC pulmonar con contraste
C. Biopsia por punción transtorácica
D. Resonancia magnética pulmonar
EXAMEN NACIONAL DE MEDICINA 2021
wwwww.qxmedic.com
www.grupoqxmedic.com
60. Lactante mujer de 11 meses nacida por cesárea
debido a presentación podálica y macrosomía.
Al examen: asimetría de pliegues en glúteos y
signo de Galeazzi positivo. ¿Cuál es el
diagnóstico probable?
A. Artritis de cadera
B. Agenesia sacra
C. Artrosis de cadera
D. Displasia de cadera
61. Mujer de 74 años, sufre de hipermetropía,
desde hace 24 horas viene presentando dolor
en el ojo izquierdo con pérdida visual
paulatina, hiperemia ocular con opacidad de
córnea y midriasis. A la palpación aumento de
la tonicidad del ojo. ¿Cuál es el probable
diagnóstico?
A. Parálisis del III par craneal
B. Glaucoma crónico
C. Uveítis
D. Glaucoma agudo
62. Mujer de 43 años llegó a Lima para trabajar
hace 3 meses y desde entonces presenta
insomnio, cambios de humor e irritabilidad.
Habla poco, es muy desconfiada, en el trabajo
se desenvuelve lo mejor que puede y se alegra
cuando escucha noticias de su terruño o de sus
familiares. ¿Qué trastorno se evidencia en este
caso?
A. Disociativo
B. Somatización
C. Adaptación
D. Ansioso-depresivo
63. Neonato a término en segundo día de vida con
Trisomía 21. Se evidencia intolerancia oral,
vómitos biliosos, abdomen moderadamente
distendido, timpánico, no signos de irritación
peritoneal. ¿Cuál sería el diagnóstico más
probable?
A. Atresia duodenal
B. Atresia esofágica
C. Estenosis hipertrófica de píloro
D. Atresia de colon distal
64. Varón de 50 años que se automedicó durante 7
días con altas dosis de aspirina, azitromicina e
ivermectina por sospecha de infección por
SARS-CoV-2. Requirió endoscopia digestiva alta
por presentar abundante sangrado. ¿Cuál es la
causa básica de esta complicación?
A. Inmunosupresión
B. Agotamiento de vitamina K
C. Lisis de eritrocitos
D. Inhibición de la ciclooxigenasa
65. Varón de 50 años acude por dolor abdominal
intenso en epigastrio con irradiación a región
lumbar que se inició hace 7 días, hace 2 días se
agrega náuseas e hiporexia. Antecedente de
alcoholismo crónico. ¿Qué examen debe
solicitar para confirmar diagnóstico?
A. Fosfatasa alcalina
B. Amilasa
C. Lipasa
D. Transaminasas
66. Un organismo no gubernamental financiará
intervenciones para la disminución de las
incidencias de embarazo en adolescentes con
la condición de que estas se reduzcan por lo
menos en 10% al año. ¿Cual es la intervención
inicial que el equipo de gestión debe asegurar
para minimizara incertidumbre y lograr el
resultado esperado?
A. Liderazgo del gobierno local
B. Calidad de información
C. Contrato de obstetras
D. Adecuación cultural del parto
67. Mujer de 50 años, con Diabetes Mellitus tipo 2
de 15 años de evolución, con mal control
metabólico. Refiere desde hace 1 año dolor y
parestesias en MMII. Al examen: pérdida de
sensibilidad al dolor en MMII. ¿Cuál es el
medicamento más apropiado para este
problema?
A. Tiamina
B. Gabapentina
C. Tramadol
D. Propanolol
EXAMEN NACIONAL DE MEDICINA 2021
wwwww.qxmedic.com
www.grupoqxmedic.com
68. Niña de 2 años, inicia hace 5 días con fiebre,
cuadro catarral intenso con rinorrea,
conjuntivitis, fotofobia y tos seca. Al examen:
manchas en la mucosa yugal, erupción
exantemática maculopapular morbiliforme
rojo vinoso y confluente generalizada en todo
el cuerpo. ¿Qué antecedente es importante
conocer para un diagnóstico probable?
A. Esquema de vacunación antisarampión
B. Si tuvo contacto con varicela
C. Si tiene alergia a medicamento o alimento
D. Si tuvo contacto con garrapatas
69. Albañil de 48 años, portador de tumoración
inguino escrotal izquierda, reductible y
asintomática. Abdomen: Tumoración blanda
de 5 cm de diámetro, no dolorosa a la
palpación. Maniobra de Valsalva (+), maniobra
de Coley (-). ¡Cuál es el probable diagnóstico?
A. Hernia crural
B. Eventración abdominal
C. Hernia inguinal directa
D. Hernia inguinal indirecta
70. Segundigesta de 28 semanas, presenta
contracciones uterinas desde hace 12 horas.
Primer hijo pretérmino nacido por cesárea. Al
examen: dinámica uterina 2 en 10 minutos,
intensidad ++ y duración de 30 segundos. Tacto
vaginal: dilatación 4 cm, borramiento 80%.
¿Cuáles el diagnóstico?
A. Pródromos de trabajo de parto
B. Labor de parto pretérmino
C. Amenaza de labor de parto pretérmino
D. Incompetencia cervical
71. Adolescente de 15 años, por primera vez ayuda
en la cosecha de uvas, presenta dolor
abdominal tipo retortijón, vómitos, diarrea y
fasciculaciones musculares. ¿Cuál es la
sospecha diagnóstica?
A. Intoxicación alimentaria
B. Hipoglicemia
C. Neurosis conversiva
D. Intoxicación por insecticidas
72. El equipo de gestión de un centro de salud I-4
encuentra que en los últimos 5 años se ha
incrementado la población adulta mayor, de
los cuales 60% son de sexo femenino, 95%
están en situación de pobreza extrema y 90%
cuentan con seguro integral de salud. ¿Cuál
fase del análisis de la situación de salud esta
gestionando?
A. Priorización de problemas
B. Identificación de actores
C. Definición de líneas de acción
D. Análisis del entorno
73. Recién nacido masculino, antecedente
obstétrico de polihidramnios. Presenta
vómitos biliosos. Al examen: abdomen
distendido, RHA incrementados. RX abdomen:
Imagen de doble burbuja. ¿Cuál es el
diagnóstico probable?
A. Atresia duodenal
B. Malrotación intestinal
C. Estenosis hipertrófica del píloro
D. Hernia diafragmática
74. Varón de 65 años, hace 5 meses fiebre
intermitente, cefalea y artralgias. Al examen:
dolor a la presión en la región temporal
bilateral, claudicación de la mandíbula y
amaurosis bilateral. Laboratorio: Hb 8.5 g/dl,
VSG 150 mm/h. ¿Cuál es la conducta
terapéutica a seguir?
A. Ciclofosfamida
B. Metrotexate
C. Ácido acetilsalicílico
D. Corticoesteroides
75. Mujer de 50 años, presenta cólicos biliares a
repetición desde hace 2 años. Hace 24 horas
intenso dolor abdominal en epigastrio con
náuseas y vómitos. Distención abdominal.
Amilasa 3000 UI/d, Lipasa 600 UI/d, Bilirrubina
totales 3 mg/dl, BD 2.5 mg/dl, BI 0,5 mg/dl.
¿Cuál examen de imagen solicitaría?
A. Tomografía abdominal
B. Colangiografía retrógrada endoscópica
C. Ecografía abdominal
D. Gammagrafía de hígado
EXAMEN NACIONAL DE MEDICINA 2021
wwwww.qxmedic.com
www.grupoqxmedic.com
76. Varón de 28 años, desde hace 3 días, fiebre,
cefalea, mialgias intensas. Antecedente de
haber visitado Iquitos hace 9 días. Al tomar la
presión arterial, aparece un gran número de
petequias en el antebrazo. ¿Cuál es el
diagnóstico más probable?
A. Malaria
B. Fiebre amarilla
C. Dengue
D. Leptospirosis
77. Adolescente de 12 años presenta cefalea desde
hace 1 mes. Se asocia tos a predominio
nocturno y rinorrea intermitente. Al examen:
secreción purulenta en la pared posterior de la
faringe. ¿Cuál es el diagnóstico a plantear?
A. Sinusitis subaguda
B. Rinitis alérgica
C. Rinofaringitis crónica
D. Rinosinusitis aguda
78. Niño de 6 años, desde hace 3 días presenta tos
productiva. Dolor retroesternal al toser y al
respirar profundamente. Al examen: roncantes
diseminados en ambos hemitórax. ¿Cuál es el
diagnóstico probable?
A. Traqueobronquitis
B. Bronquiolitis aguda
C. Bronquitis aguda
D. Neumonía intersticial
79. Varón de 65 años, desde hace 8 semanas
refiere llenura precoz, disminución de peso y
distención abdominal. Al examen: adelgazado,
pálido, se palpa adenopatía supraclavicular
izquierda de consistencia pétrea y abdomen
globuloso con signo de oleada positivo.
Laboratorio: anemia microcítica e hipocrómica.
¿Qué examen es necesario para proponer un
diagnóstico?
A. Biopsia ganglionar
B. Paracentesis exploratoria
C. Endoscopia digestiva alta
D. Colonoscopía virtual
80. Mujer de 34 años, consulta por baja de peso de
aproximadamente 6 kilos en un mes. Refiere
incremento de apetito y sed, intolerancia al
calor y sudoración excesiva. Al examen: PA
120/50 mmHg, FC 120 x', FR 16 x', T 37,5 C piel
húmeda y caliente. ¿Cuál es el diagnóstico
probable?
A. Adenopatía hipofisiario
B. Hipertiroidismo
C. Feocromocitoma
D. Diabetes tipo 2
81. Varón de 13 años, desde hace tres días
presenta dolor en rodilla derecha que le impide
caminar y fiebre de 38,5 C. Al examen: rodilla
derecha dolorosa, aumentada de volumen y
con signos inflamatorios. Hemograma con
leucocitos de 25000 x mm³ y 10% de
abastonados. ¿Cuál es el probable germen
responsable de la infección?
A. Staphylococcus aureus
B. Escherichia coli
C. Corynebacterium pyogenes
D. Haemophilus influenzae
82. Chofer de ambulancia colisiona contra un poste
y es conducido a Emergencia con fractura
expuesta de fémur derecho. Al examen: PA
100/60 mmHg, herida en muslo con exposición
de plano muscular. RM: fémur con trazo de
fractura diafisaria no desplazada. ¿Cuál es el
manejo inicial que indicaría?
A. Colgajo muscular
B. Reducción cruenta
C. Desbridamiento
D. Injerto cutáneo
83. Mujer de 64 años VIH (+), desde hace 3 días
dolor urente en la región lateral de hemitórax
izquierdo, seguido de eritema y vesículas de
tamaño variable a lo largo del dermatoma. Test
de Tzanck: cambios citoplasmáticos típicos y
presencia e células gigantes multinucleadas.
¿Cuál es el diagnóstico más probable?
A. Herpes zóster
B. Dermatitis de contacto
C. Pioderma gangrenoso
D. Dermatosis neutrofílica
EXAMEN NACIONAL DE MEDICINA 2021
wwwww.qxmedic.com
www.grupoqxmedic.com
84. Gestante de 33 semanas de embarazo sin
control prenatal, acude por sangrado vaginal,
no refiere otras molestias. Cesareada 1 vez
hace 3 años. Al examen: PA 100/60 mmHg, FC
86 x', AU 30 cm, feto en LCI, LCF 130 x'
presentación flotante a la cuarta maniobra de
Leopold. No se constataron contracciones
uterinas. ¿Qué procedimiento deberíamos
evitar en esta paciente?
A. Ecografía obstétrica
B. Tacto vaginal
C. Especuloscopía
D. Ecografía transvaginal
85. Puérpera mediata de 3 días por cesárea. Inicia
con fiebre 38.5 C en 2 horas separadas. Al
examen: se palpa útero doloroso a 2 cm de la
cicatriz umbilical. A la especuloscopía:
secreción purulenta. ¿Cuál es la conducta a
seguir?
A. Explorar herida operatoria
B. Solicitar ecografía transvaginal
C. Preparación para legrado puerperal
D. Iniciar antibioticoterapia
86. Varón 83 años acude por sangrado nasal
severo. Examen: funciones vitales estables,
palidez leve de piel y mucosas. Se observa
sangrado en regular cantidad por fosa nasal
derecha y por boca. ¿Cuál es el procedimiento
de elección?
A. Taponamiento posterior
B. Taponamiento Anterior
C. Cauterización del vestíbulo nasal
D. Compresión nasal externa
87. Mujer de 36 años con antecedentes de litiasis
urinaria de 5 años. Desde hace 30 días dolor
lumbar derecha y fiebre continua. Al examen:
PPL derecha +++. Laboratorio: leucocitosis
14500/mm³ sin desviación izquierda, Hb 9,5
g/dl y VSG 92 mm/h. Sedimento urinario:
leucocitos 50 x campo y la citología urinaria
muestra abundantes macrófagos con
aspecto espumoso. ¿Cuál es el diagnóstico más
probable?
A. Adenocarcinoma renal
B. Pielonefritis xantogranulomatosa
C. Absceso perinéfrico
D. Pielonefritis enfisematosa
88. Mujer de 30 años, puérpera de 2 semanas.
Consulta por fiebre. Al examen: dolor,
enrojecimiento e induración de piel en mama
derecha. ¿Cuál es la actitud a seguir?
A. Antibiótico y seguimiento clínico
B. Mamografía de mama
C. Biopsia con aguja gruesa
D. Drenaje y control en 2 semanas
89. Mujer de 20 años, acude al servicio de
emergencia porque hace 2 horas se quemó con
agua hervida. Al examen: piel con ampollas y
flictenas que abarcan toda la extremidad
superior derecha. El compromiso de superficie
corporal corresponde al _____ %.
A. 9
B. 27
C. 4.5
D. 18
90. Tercigesta de 39 semanas por FUR en labor de
parto, con dolor abdominal súbito y sangrado
vaginal. Cesareada anterior con periodo
intergenésico corto. Al examen: se palpan
partes fetales en abdomen. Tacto vaginal:
dilatación 3, estación -3, membranas rotas con
líquido sanguinolento. Latidos fetales 100 x'.
¿Cuál es el diagnóstico?
A. Desprendimiento prematuro de placenta
B. Rotura uterina
C. Rotura de vasa previa
D. Placenta previa
91. Un centro de salud reporta aumento en la
incidencia de tuberculosis pulmonar BK
positivo. El análisis de la situación de salud del
ámbito muestra hacinamiento y aumento del
desempleo ¿Cuál de los siguientes aspectos es
el de mayor influencia en este problema?
A. Limitada búsqueda de sintomáticos
B. No uso de mascarilla
C. Determinantes sociales
D. Multidrogoresistencia
EXAMEN NACIONAL DE MEDICINA 2021
wwwww.qxmedic.com
www.grupoqxmedic.com
92. Varón de 25 años consulta por infertilidad. Los
exámenes auxiliares evidencian un problema
de maduración de los espermatozoides. Los
exámenes de la esposa resultaron normales. La
explicación a esta situación se debe a
disfunción del/de la……
A. cuerpo del epidídimo
B. conducto eyaculador
C. vesícula seminal
D. conducto deferente
93. Mujer de 22 años acude porque súbitamente
presenta dolor lumbar izquierdo irradiado
hacia región inguinal del mismo lado que no
calma con analgésicos. Al examen PPL
izquierdo +++ Examen de orina: hematíes 20 a
50 x campo, cilindros eritrocitarios y cristales
de oxalato. ¿Cuál es el probable diagnóstico?
A. Cistitis aguda
B. Litiasis vesical
C. Litiasis renal
D. Uretritis
94. Lactante de 5 meses de edad, durante su
primer control en un establecimiento de salud
1-2 se detectó una ganancia de peso mensual
menor de 500 g. No fue llevado a sus citas
posteriores. ¿Qué acción se debe priorizar?
A. Denunciar a los padres
B. Visita domiciliaria
C. Insistir en programación mensual
D. Referencia a tercer nivel de atención
95. Mujer de 16 años con historia de menstruación
prolongada hasta 15 días. Al examen: se
detecta palidez, Hb 8 g/dl; pruebas para
hemofilia deficiencia importante del factor von
Willebrand. ¿Cuál es la terapia más adecuada?
A. Transfusión de plaquetas
B. Plasma fresco congelado
C. Plasma residual
D. Transfusión de sangre total
96. Un médico serumista , jefe de un
establecimiento de salud 1-4, requiere
presentar el plan operativo anual ¿Con cuál de
los siguientes procesos debe iniciar la
elaboración del plan?
A. Formulación de metas y objetivos
B. Determinación de los problemas sanitarios
C. Determinación de actividades
D. Asignación de recursos
97. Multípara de 41 semanas acude por
contracciones uterinas 3/10 minutos, 40
segundos de duración, intensidad +++/+++, con
placenta de inserción baja por ecografía y que
sangra en escasa cantidad. Tacto vaginal:
dilatación 7, altura de presentación -1,
membranas integras, LCF 148 x'. ¿Cuál sería el
manejo adecuado?
A. Preparar a la paciente para operación cesárea
B. Poner vía endovenosa, romper bolsas y esperar
el parto vaginal
C. Hospitalización y conducta expectante
D. Acentuación del parto con oxitocina
98. Varón de 65 años, traído a Emergencia por
sincope de esfuerzo. Al examen: PA 130/80
mmHg, despierto, pulso irregular. EKG (ver
imagen). ¿Cuál es el diagnóstico más probable?
A. Insuficiencia aórtica
B. Miocardiopatía hipertrófica
C. Taponamiento cardiaco
D. Bloqueo A-V completo
99. Mujer de 54 años, reiteradamente acude a
Emergencia por cefalea pulsátil, náuseas y
vómitos En la anamnesis: habita en ambiente
cerrado y refiere usar estufa de gas. Al examen:
PA 110/60 mmHg. FC 120 X’ , FR 26 x’, SaO2
95% a aire ambiental. Desorientada, confusa e
intensa debilidad muscular. ¿Cuál es la terapia
más adecuada?
A. Óxido nitroso en inhalación
B. Intubación endotraqueal
C. Oxígeno al 100%
D. Ventilación a alto flujo a 30 It/min
EXAMEN NACIONAL DE MEDICINA 2021
wwwww.qxmedic.com
www.grupoqxmedic.com
100. Niño de 10 años, desde hace 4 días presenta
fiebre dolor abdominal vómitos y coluria
Antecedente: consume alimentos en el quiosco
escolar. Al examen: ictericia y hepatomegalia.
Laboratorio: TGO 740, TGP 1020. Tiempo de
protrombina normal. ¿Cuál es el manejo
apropiado?
A. Gammaglobulina especifica y sintomáticos
B. Hepatoprotectores y fitomenadiona
C. Hidratación y sintomáticos
D. Administrar Interferón gamma a dosis
terapéutica
101. Adolescente de 13 años, hospitalizada por
deshidratación y desnutrición con IMC 15. La
madre refiere que con frecuencia su hija se
provoca vómitos. ¿Qué trastorno del medio
interno espera encontrar?
A. Acidosis metabólica
B. Acidosis hipoclorémica
C. Alcalosis hiperclorémica
D. Alcalosis hipoclorémica
102. Varón de 24 años acude por fiebre y artralgias.
Al examen: acné severo con diferentes
lesiones, incluso cicatriciales, en cara y tórax.
Hemograma: leucocitosis. ¿Cuál es el tipo de
acné mas probable?
A. Pustuloso
B. Fulminante
C. Nodular
D. Vulgar
103. Varón de 45 años, obeso acude a emergencia
por presentar, dolor abdominal al epigástrico
irradiado a la espalda, vómitos biliosos
incoercibles. Al examen FC: 120 x’, PA 80/60
mmHg, soporosos, dolor abdominal difuso,
ictericia. Triglicéridos elevados, Amilasa
normal. ¿Cuál es la conducta inicial a seguir?
A. Hidratación EV agresiva
B. Alimentación enteral
C. CPRE urgente
D. Antibiótico profilaxis
104. Primogénito de un mes de edad, llevado a
Emergencia por haber presentado vómitos en
proyectil, no bilioso. Al examen: deshidratación
moderada, se palpa tumoración pequeña en
epigastrio. RX abdomen simple: burbuja de gas
en el estómago. ¿Cuál es el diagnóstico?
A. Estenosis yeyunal
B. Estenosis duodenal
C. Hipertrofia de píloro
D. Gastrosquisis
105. Varón de 70 años con antecedente de
alcoholismo, sufre caída desde una altura de 5
metros ingresa a Trauma Shock con Glasgow
10. Tomografía cerebral con Imagen
refringente en forma de medialuna, con bordes
poco definidos. ¿Cuál es el tratamiento
indicado?
A. Administrar vitamina K.
B. Administrar coagulantes
C. Craneotomía con drenaje
D. Derivación ventrículo atrial
106. Lactante de 2 años desde hace 5 días presenta
tos exigente Hace un día se agrega fiebre y
respiración rápida. Al examen FC 172 X’ , FR 50
X’ T° 40 °C Sao2 96% tirajes subcostal e
intercostal moderados. Hemitórax derecho
amplexación y murmullo vesicular disminuidos
y crepitantes diseminados. La principal
sospecha es neumonía …
A. adquirida en la comunidad
B. atípica
C. complicada con efusión
D. complicada con empiema
107. Mujer de 38 años, asmática, acude a
emergencia por inicio súbito de disnea y tos. Al
examen obesidad centrípeta FC 120 x’ FR 30x’
SaO2 90 % confusa responde con frases cortas.
A la auscultación sibilantes diseminados en
ambos hemitórax ¿Cuál es la conducta
inmediata a seguir?
A. Intubación orotraqueal
B. Administración de corticoides
C. Nebulización con B2 agonistas
D. Infusión de sulfato de magnesio
EXAMEN NACIONAL DE MEDICINA 2021
wwwww.qxmedic.com
www.grupoqxmedic.com
108. Neonato a término hospitalizado con
diagnóstico de periodo de adaptación
prolongado, con indicación de suplemento de
oxígeno mínimo continuo. Hemoglucotest 45
mg/dl ¿Cual es la conducta a seguir?
A. Iniciar bolo dextrosa al 10%
B. Iniciar tratamiento antibiótico
C. Reevaluación del hemoglucotest
D. Restringir vía oral
109. Paciente agredido con un objeto penetrante a
nivel del quinto espacio intercostal línea media
clavicular izquierda. Funciones vitales PA 90/60
mmHg, FC 110 X’ , PVC 16 mmHg . Al examen
ingurgitación yugular izquierda y ruidos
cardiacos apagados ¿cuál es la conducta
inmediata a seguir?
A. Pericardiocentesis
B. Fluidoterapia con cristaloides
C. Pase a cuidados intensivos
D. Drenaje pleural izquierdo
110. Recién nacido a término, a las 3 horas de vida
presenta cianosis perioral y ungueal. La
administración de oxigeno no mejora el color
de la piel ni la saturación. ¿Cuál es el
diagnostico probable?
A. Persistencia del conducto arterioso
B. Comunicación interventricular
C. Estenosis pulmonar
D. Comunicación interauricular
111. Varón de 70 años hipertenso antiguo, evaluado
en Centro de salud por inicio reciente de disnea
cansancio y fatiga. Al examen FR 40 x’ , FC
120x’, pa 130/70 mmHg, no tolera el decúbito,
ingurgitación yugular + + +, crepitantes y
sibilancias en ambos campos pulmonares. La
conducta a seguir es:
A. Diuréticos vía endovenosa
B. Inhibidores de la enzima convertidora de la
angiotensina
C. Referencia inmediata al establecimiento de
mayor complejidad
D. Bloqueadores Beta adrenérgicos
112. ¿Qué principio bioético se vulnera cuando se
brinda asistencia mecánica ventilatoria a un
paciente adulto , que previamente había
documentado su deseo de no ser intubado ni
conectado a un ventilador mecánico
A. No maleficencia
B. Justicia
C. Beneficencia
D. Autonomía
113. Mujer de 50 años se resbala y al caerse el peso
de su cuerpo es amortiguado por su
extremidad superior izquierda. Al examen
deformación y aumento de volumen de brazo
izquierdo, pulsos presentes. RX: fractura del
tercio medio de la diáfisis del húmero ¿Qué
manifestación clínica se debe presentar?
A. Mano péndula
B. Parestesia hipotenar
C. Mano en garra
D. Llenado capilar prolongado
114. Se desea hacer una investigación sobre el
imparto de la cuarentena por COVID-19 en el
estado nutricional de estudiantes de medicina
de una universidad pública. El marco
conceptual de la investigación debe incluir
prioritariamente.
A. problemas de conectividad en la educación
virtual
B. Niveles de estrés educativo en universitarios
C. Criterios de estilos de vida saludable en adultos
jóvenes
D. Secuela por confinamiento
115. Lactante de 1 año traído por haber presentado
rigidez con relajación de esfínteres que duró
unos minutos. No presenta antecedentes
patológicos. Al examen Fc 120 X’ t38 °C,
despierto hipoactivo, nos signos de meníngeos,
tono muscular y reflejos normales ¿Cuál es el
manejo inmediato?
A. Bajar la temperatura
B. Interconsulta a neurología
C. Administración anticonvulsivantes
D. Realizar electroencefalograma
EXAMEN NACIONAL DE MEDICINA 2021
wwwww.qxmedic.com
www.grupoqxmedic.com
116. Mujer de 22 años desde hace 1 hora presenta
sensación de falta de aire, sudoración,
palpitaciones , dolor torácico opresivo y
temblor generalizado que la hacen sentir “ que
se va a morir”. El cuadro cede
espontáneamente. Refiere que el episodio
suele repetirse de forma esporádica ante
situaciones de estrés ¿Cuál es el diagnóstico
más probable?
A. Crisis de pánico
B. Depresión mayor
C. Trastorno obsesivo
D. Síndrome de abstinencia
117. Varón de 69 años con diagnostico de
pancreatitis crónica y alcoholismo crónico,
desde hace 6 meses muestra aumento en la
frecuencia de deposiciones las cuales son de
consistencia semisólida, de aspecto pálido,
brillantes y untuosas. ¿Cuál es el diagnostico
más probable?
A. Insuficiencia pancreática exocrina
B. Síndrome Zollinger-Ellison
C. Ampuloma
D. Gastritis atrófica
118. Agricultor de 40 años , acude a centro de salud
por presentar hace 4 días fiebre, cefalea
artralgias y mialgias especialmente en
pantorrillas . Refiere exposición a canales de
regadío durante el último mes ¿Cuál es la
clasificación del caso?
A. Probable leptospirosis
B. Sospecha de peste
C. Confirmado de Dengue
D. Contacto de fiebre amarilla
119. Médico infectólogo de 45 años, desde hace 3
meses se ha aislado de sus compañeros de
trabajo. Refiere que ha inventado una vacuna
contra el coronavirus y que por eso le darán el
Premio Nobel. Además, expresa que le quiere
robar su invento y que quieren matarlo. Cree
que le han puesto micrófonos y cámaras en su
departamento. Al examen: desaseado y
ansioso. ¿Cuál es su diagnóstico probable?
A. Sindrome de Burnout
B. Psicosis depresiva
C. Esquizofrenia paranoide
D. Trastorno Bipolar
120. Mujer de 30 años, desde hace 3 meses
presenta episodios de palpitaciones, cefalea e
hiperhidrosis. Al examen: PA 180/120 mmHg,
FC 100 x', piel pálida, fría y sudorosa; ruidos
cardiacos de buena intensidad. Laboratorio:
elevación de catecolaminas y metanefrinas
urinarias. ¿Cuál es el diagnóstico más
probable?
A. Hipertensión renovascular
B. Feocromocitoma
C. Hiperaldosteronismo
D. Hipertensión arterial esencial
121. Lactante de 10 meses, es traído a Emergencia
por presentar temperatura de 39.5 C. La madre
refiere que presentó retroversión ocular,
movimientos generalizados con relajación de
esfínteres y pérdida de la conciencia durante
30 minutos. Luego despertó, lactó durante 5
minutos y quedó dormido. ¿Cuál es el
diagnóstico probable?
A. Convulsión febril simple
B. Epilepsia parcial continua
C. Estatus epiléptico febril
D. Convulsión febril compleja
122. Primigesta de 33 semanas por FUR, acude al
centro de salud donde usted está haciendo su
SERUMS. Refiere pérdida de líquido hace 8
horas que mojó hasta las rodillas. Signos vitales
normales. No se evidencian contracciones
uterinas. Al tacto vaginal el cuello uterino no
está dilatado, pero sí reblandecido y el líquido
amniótico es claro. Latidos fetales 154 x'. ¿Cuál
es el manejo más adecuado?
A. Tocolisis, hidratación y referencia a hospital de
mayor complejidad
B. Maduración pulmonar, profilaxis antibiótica y
referencia a hospital de mayor complejidad
C. Neuroprotección, maduración pulmonar y
reevaluación en 4 horas
D. Profilaxis antibiótica y evolución espontánea
de trabajo de parto
EXAMEN NACIONAL DE MEDICINA 2021
wwwww.qxmedic.com
www.grupoqxmedic.com
123. Mujer de 61 años con EPOC acude a
emergencia por presentar mayor disnea,
incremento de tos y de esputo RX de tórax:
signos de enfisema sin opacidades focales.
Laboratorio: PaO2 62mmHg. ¿Cuál es la causa
de la hipoxemia?
A. Disminución del espacio muerto
B. Aumento del shunt fisiológico
C. Desequilibrio ventilación-perfusión
D. Deterioro de la difusión
124. Varón de 45 años, sometido a un examen de
retina, luego del cual presenta fotofobia y
visión borrosa. Pupilas: midriasis bilateral, no
reactivas a la luz ni a la acomodación. No otro
trastorno oculomotor. ¿Cuál es el diagnóstico
más probable?
A. Midriasis farmacológica
B. Parálisis del III par craneal
C. Pupilas de Argyll Robertson
D. Síndrome de Horner
125. Varón de 35 años, consulta por nauseas, cólico
abdominal y diarreas posterior a la ingesta de
leche y otros productos lácteos. Examen de
heces: azúcares reductores ++. La causa más
probable es
A. Deficiencia de lactasa intestinal
B. Deficiencia de amilasa pancreática
C. Intolerancia a la proteína de la leche
D. Deficiencia de la sacarasa-isomaltasa intestinal
126. Varón de 45 años, acude a consultorio por
presentar desde hace un mes dolor tipo urente
en epigastrio que aparece 5 a 6 horas después
de la ingesta de alimentos, ocasionalmente el
dolor lo despierta por las noches. Niega
pérdida de peso y disfagia. Sin antecedentes
relevantes. Al examen: no evidencia hallazgos
significativos. ¿Cuál sería el diagnóstico más
probable?
A. Gastritis aguda
B. Reflujo gastroesofágico
C. Úlcera péptica
D. Cáncer de estómago
127. Varón de 60 años, reside en Cerro de Pasco,
consulta por cefalea, mareos y acúfenos desde
hace 2 años, con empeoramiento en el último
mes. Al examen: buena nutrición, hiperemia
conjuntival, cianosis de dedos y labios.
Laboratorio: Hb 22 g/dl Hto 66% y SaO2 88%
¿Cuál es el diagnóstico probable?
A. Policitemia vera
B. Mal de montaña crónico
C. Hipoxemia refractaria
D. Policitemia fisiológica secundaria
128. Escolar de 7 años con odinofagia y fiebre desde
hace 3 días. Al examen: adenopatías cervicales
dolorosas, orofaringe con tractos
blanquecinos. ¿Cuál es el agente causal más
frecuente?
A. Adenovirus
B. Streptococcus grupo B
C. Streptococcus beta hemolítico grupo A
D. Moraxella catarrhalis
129. Varón de 24 años, refiere contacto sexual
casual sin protección hace 4 días, acude por
presentar secreción mucopurulenta por la
uretra asociada a disuria. ¿Cuál es el
tratamiento empírico de elección?
A. Penicilina G sódica + gentamicina
B. Ceftriaxona + azitromicina
C. Clindamicina + amikacina
D. Penicilina benzatínica + clindamicina
130. Recién nacido de 36 semanas de edad
gestacional y 12 horas de vida que inicia cuadro
de dificultad respiratoria leve e hipotermia.
Antecedente materno de RPM mayor de 18
horas. ¿Cuál es el diagnóstico más probable?
A. Enfermedad de membrana hialina
B. Taquipnea transitoria del recién nacido
C. Hemorragia intraventricular
D. Neumonía neonatal
131. Según el calendario de vacunación MINSA, ¿a
que edad en años debe administrarse la dosis
de refuerzo de la vacuna de varicela?
A. 4
B. 10
C. 6
D. 5
EXAMEN NACIONAL DE MEDICINA 2021
wwwww.qxmedic.com
www.grupoqxmedic.com
132. Lactante de 9 meses con historia de diarreas
frecuentes. Hace 3 días presenta vómitos y
diarreas líquidas 4-6 veces al día. Al examen:
peso 5 kg, pulso débil, hipoactivo, fascie de
anciano, TCSC muy disminuido, llenado capilar
> 3 segundos, ojos hundidos: AGA: pH 7.2,
pCO2 25, HCO3 10 mEq/L. ¿Cuál es la condición
asociada a su estado nutricional que empeora
el pronóstico?
A. Hiperkalemia
B. Hipomagnesemia
C. Hipocalcemia
D. Hiponatremia
133. Varón de 32 años, con diagnóstico de Giardiasis
intestinal, en el cuarto día de tratamiento con
metronidazol 500 mg dos veces al día, luego de
participar en una reunión social donde ingiere
licor, lo llevan a la emergencia por presentar
dificultad respiratoria, náuseas, vómitos,
palpitaciones y excitación psicomotriz. ¿Cual es
el diagnóstico más probable?
A. Efecto antabus.
B. Ingesta excesiva de alimentos grasos
C. Excesiva dosis de metronidazol
D. Toxicidad por alcohol
134. Lactante de 5 meses con pobre ganancia
ponderal. Polipneico y diaforético durante la
lactancia. Al examen: taquicárdico, soplo
sistólico II/IV en mesocardio, borde hepático a
3 cm debajo del reborde costal. ¿Cuál es el
diagnóstico probable?
A. Insuficiencia cardiaca
B. Ducto arterioso persistente
C. Miocarditis viral
D. Enfermedad de Kawasaki
135. Pretérmino de 35 semanas de edad gestacional
y 2 horas de vida. Peso al nacer 2000 g.
Presenta irritabilidad, temblor, dificultad en la
succión y 30 minutos después se torna
letárgico. Hemoglucotest: 30 mg/dl. Se decide
colocar dextrosa al ……% 200 mg/kg, seguido
por una infusión de 6-8 mg/kg/min.
A. 50
B. 10
C. 33
D. 5
136. Varón de 58 años, acude por haber presentado
convulsión tónico clónica generalizada con
relajación de esfínteres que duró
aproximadamente 30 segundos. Al examen se
encuentra algo confuso y no recuerda bien lo
sucedido. No tiene antecedentes de anteriores
convulsiones. ¿Cuál es el examen auxiliar
preferencial para confirmar el diagnóstico?
A. Resonancia magnética de cerebro
B. Electroencefalograma
C. Punción lumbar
D. Tomografía computarizada de cerebro
137. Chef de 30 años sufre quemaduras de 2do y 3er
grado en cara y cuello. Al examen: edema de
mucosa oral, esputo de color negro, restos
carbonizados en boca y fosas nasales, estridor
laríngeo y cianosis. ¿Cuál es la acción
prioritaria?
A. Intubación endotraqueal
B. Radiografia de tórax
C. Tomografía cervical
D. Corticoides EV
138. Gestante de 28 semanas acude con dolor tipo
contracción persistente y ginecorragia. Refiere
que no siente movimientos de su bebé, G2
P1001, Cesárea hace 10 meses por causa
similar ala gestación actual. Al examen: PA
130/90 mmHg, FC 100 x¨, FR 16 X', LCF 102 x.
¿Cuál sería el factor principal que condiciona la
complicación del actual embarazo?
A. Periodo intergenésico corto
B. Trastorno hipertensivo del embarazo
C. Placenta previa
D. Cesárea anterior
139. Varón de 45 años, estando en Emergencia
presenta súbita pérdida de conciencia con
ausencia de pulso y apnea. En el monitor
cardíaco se evidencia ritmo acelerado (ver
imagen). ¿Cuál es la mejor opción terapéutica?
A. Desfibrilación
B. Cardioversión
C. Ablación
D. Betabloqueante EV
EXAMEN NACIONAL DE MEDICINA 2021
wwwww.qxmedic.com
www.grupoqxmedic.com
140. Varón de 60 años con antecedente de colitis
ulcerosa, desde hace una semana presenta
diarreas, indicándose con loperamida sin éxito.
Al examen: FC 120 x’, T 39 °C. soporoso,
distensión abdominal con RHA ausentes.
Hemograma Leucocitosis con neutrofilia Hb 7
gr/dl. ¿Cuál es el diagnóstico más probable?
A. Estenosis sigmoidea
B. Isquemia mesentérica
C. Diverticulitis aguda
D. Megacolon tóxico
141. Neonato del 14 días acude a control de niño
sano. Al examen buen estado general sin
fiebre, cordón umbilical no maloliente, sin
secreciones y con restos de nitrato de plata.
Resto del examen sin alteraciones.
Antecedentes no relevantes. Pruebas
metabólicas normales. ¿Cuál es el diagnóstico
probable?
A. Problemas de inmunidad humoral
B. Retraso secundario a problemas del uraco
C. Retraso de caída del cordón sin patología
asociada
D. Retraso por defecto congénito de la
adhesividad leucocitaria
142. Mujer 35 años, con artritis de rodilla izquierda
el líquido sinovial con tinción de Gram,
demuestran bacterias grampositivas
agrupadas en racimos de uva. ¿Cuál es la mejor
conducta a seguir?
A. Drenaje y antibióticos vía parenteral
B. Antibióticos vía parenteral con
antinflamatorios
C. Drenaje y antinflamatorios parenterales
D. Lavado intra articular continuo
143. Gestante de 24 semanas, asintomática, con
prueba de tuberculina positiva y RX de tórax
normal. Esposo acaba de ser diagnosticado de
TBC, BK Positivo. ¿Cuál es la conducta a seguir?
A. Radiografía en seis meses
B. TEM de tórax en un mes
C. Isoniacida profiláctica
D. BK seriado por tres tomas
144. Mujer de 25 años con episodios agudos de
migraña a razón de más de 4 episodios por
mes. ¿Cuál sería la mejor medida terapéutica
para evitar la recurrencia de los episodios?
A. Tramadol
B. Sumatriptan
C. Propanolol
D. Ibuprofeno
145. Segundigesta de 14 semanas con sobrepeso,
acude a su primer control prenatal (CPN). Su
primer hijo nació con 4400 g de peso. Se le
realiza el test de detección de diabetes con 50
g de azúcar, saliendo negativo. ¿Cuál es la
conducta a seguir?
A. Repetir la prueba entre las 24 y 28 semanas
B. Solicitar un test de tolerancia a la glucosa
C. Medir glucosa cada tres semanas
D. Realizar CPN normal
146. Pre-escolar de 3 años con familia en extrema
pobreza hace 3 meses presenta fatiga,
anorexia, náuseas, dolor abdominal, diarreas y
calambres. Recibe tratamiento variados sin
mejoría. Antecedentes: no recibió lactancia
materna. Dieta pobre en carnes. Al examen:
muy adelgazado, edematoso, pálido y presenta
marcada disminución de los reflejos tendinosos
profundos. ¿Qué déficit vitamínico es el más
probable?
A. Riboflavina
B. Tiamina
C. Piridoxina
D. Niacina
147. Varón de 38 años, con IMC de 38, obeso desde
los 17 años. Ha probado múltiples tratamientos
sin éxito, pierde peso y posteriormente lo
recupera. comienza a automedicarse con
Orlistat (inhibidor de absorción de grasa),
consulta sobre qué otro tratamiento podría
adicionar. Su recomendación sería:
A. Dieta vegana
B. Metformina
C. Dieta mediterránea
D. Ejercicios aeróbicos
EXAMEN NACIONAL DE MEDICINA 2021
wwwww.qxmedic.com
www.grupoqxmedic.com
148. Niño de 6 años, llevado por su madre por
referir dolor en oído desde hace 3 horas.
Otoscopía: insecto vivo que se mueve en el
canal auditivo externo. ¿Cuál es el manejo
inicial?
A. Lavado de oído con solución salina
B. Extracción instrumental con pinza
C. Instilación ótica de aceite mineral
D. Irrigación ótica con agua oxigenada
149. Primigesta de 40 semanas en pródromos de
labor, acude por disminución de movimientos
fetales. Se realiza NST no reactivo y variabilidad
disminuida. Perfil biofísico (PBF): normal ¿Cuál
es el siguiente paso a seguir?
A. Cesárea de emergencia
B. Test estresante
C. Repetir NST en 6 horas
D. Repetir PBF en 72 horas
150. Durante su rotación de ginecología y
obstetricia, usted observó un aumento
significativo de casos de endometritis
puerperal. ¿Qué medida ayudaría a prevenir
estas infecciones?
A. Usar sonda Foley durante el parto
B. Fenobarbital
C. Disminuir los tactos vaginales innecesarios
D. Rasurar el vello púbico a todas las gestantes
151. Recién nacido a término nacido por cesárea
debido a bradicardia fetal. A las 2 horas de vida
presenta movimientos tónico clónicos
generalizados. ¿Cuál es el medicamento de
elección?
A. Levetiracetam
B. Fenobarbital
C. Difenilhidantoina
D. Midazolam
152. Varón de 20 años sufre quemadura con agua
caliente en el antebrazo derecho. Al examen:
lesiones eritematosas, ampulares y dolorosas
que palidecen a la presión. ¿Qué medida
terapéutica está indicada inicialmente?
A. Antibióticos sistémicos
B. Aplicar antibióticos tópicos
C. Vacuna antitetánica
D. Reposición hídrica según fórmula de Parkland
153. Varón de 17 años con antecedente de rinitis
alérgica, consulta por dermatitis atópica ¿Cuál
es la terapia tópica recomendada?
A. Higiene constante (≥3 veces por día)
B. Aplicación de emolientes e hidratantes
C. Corticoides tópicos de alta potencia
D. Neomicina en ungüento
154. Enfermero que labora en UCI, desde hace 5
días presenta fiebre, tos, cefalea, mialgia y
odinofagia asociadas a debilidad general. Al
examen: FR 22 x', SaO2 91% ¿Qué
característica biológica corresponde al agente
causal de esta enfermedad?
A. Se une al receptor ACE 2 a través de la proteína
S
B. Su material genético ingresa a la célula como
DNA
C. Su replicación es controlada por el DNA del
neumocito
D. La proteína M le da su morfología típica
155. Mujer del 32 años, sufre contusión grave
toraco abdominal. Al examen: equimosis en
tórax anterior bajo y epigastrio, palidez intensa
de piel y mucosas, compromiso de conciencia,
ingurgitación yugular marcada. PA 60/30
mmHg, pulso imperceptible, latidos cardiacos
poco audibles SaO2 89% Abdomen no
distendido, difícil evaluar por estado de
conciencia. EKG: complejos QRS de bajo
voltaje en todas las derivaciones. Se intenta
pericardiocentesis sin éxito. ¿Cuál es el
procedimiento prioritario a realizar?
A. Toracotomía exploratoria
B. Toracoscopía diagnóstica
C. Ventana pericárdica
D. Decorticación pericárdica
156. Paciente mujer de 35 años de edad que trabaja
en ventas, desde hace un año presenta 3 a 4
veces al mes episodios de dolor cólico
abdominal y borborigmos que ceden con la
defecación, acompañado de distensión
abdominal ocasional y alternancias de diarrea
y estreñimiento. El examen físico es normal.
¿Cuál es su impresión diagnóstica?
A. Colon irritable
B. Distensión abdominal funcional
C. Dispepsia funcional
D. Colecistitis crónica
EXAMEN NACIONAL DE MEDICINA 2021
wwwww.qxmedic.com
www.grupoqxmedic.com
157. En un colegio secundario el 80% de alumnos
del último año presentan sobrepeso u
obesidad. Usted es el médico responsable de la
estrategia sanitaria de enfermedades no
transmisibles del establecimiento de salud.
¿Qué tipo de intervención inmediata
priorizará?
A. Programación de consultas nutricionales
B. Comunicación educativa grupal
C. Clausura del quiosco escolar
D. Difusión radial de factores protectores
158. Mujer de 48 años, que hace 5 días presenta
dolor en epigastrio luego de la ingesta de
comidas grasas, 24 horas después aparece
coloración amarillenta en piel y escleras.
Colangioresonancia: múltiples cálculos en la vía
biliar principal, uno grande de 1.3 cm de
diámetro impactado en colédoco distal. ¿Cuál
es la conducta terapéutica a seguir?
A. Papilotomía endoscópica
B. Colecistectomía
C. Exploración de vía biliar y drenaje Kherr
D. Colecistostomía
159. Recién nacido, en su evaluación urogenital se
encuentra pene curvo con fimosis, orificio
uretral que desemboca en región ventral del
cuerpo del pene ¿Qué se esperaría encontrar
en el examen escrotal?
A. Varicocele
B. Bolsa escrotal vacía
C. Agenesia de epidídimo
D. Testículos normales
160. Varón de 45 años con colitis ulcerativa de 3
años de evolución. Hospitalizado por
exacerbación de la colitis, consunción, dolor
abdominal y apatía. Perfil tiroideo, T3 total y
libre disminuido, T4 libre y TSH normal. ¿Cuál
es el diagnóstico más probable?
A. Hipotiroidismo primario
B. Sindrome del eutiroideo enfermo
C. Hipotiroidismo subclínico
D. Hipertiroidismo subclínico
161. Mujer de 60 años, obesa, refiere prurito vulvar
y escaso flujo vaginal. Al examen: vulva
congestiva, eritematosa, con sequedad de sus
paredes. ¿Cuál es el diagnóstico más probable?
A. Vulvovaginitis inespecífica
B. Vulvovaginitis candidiásica
C. Vaginosis bacteriana
D. Vaginitis atrófica
162. Paciente de 25 años asintomática. En una
ecografía de rutina presenta en ovario derecho
masa quística unilocular de 4 cm sin tabiques ni
excrecencias. ¿Cuál es la conducta a seguir?
A. Quistectomía
B. Observación
C. Solicitar alfa feto proteína
D. Prescribir anticonceptivos orales
163. Varón de 38 años, refiere intenso dolor ocular,
lagrimeo y enrojecimiento mientras trabajaba
con una amoladora eléctrica sin contar con
protector facial. Al examen: cuerpo extraño
metálico que atraviesa córnea, iris y cristalino.
¿Cuál es el manejo inicial?
A. Colirio analgésico
B. Analgesia endovenosa
C. Parche compresivo
D. Tonometría
164. Primigesta de 33 semanas, consulta por cefalea
de un día de evolución. Al examen: PA 170/110
mm Hg, Hb 10.5 g/dl, plaquetas 95000/mm3,
TGO 400, TGP 300 y LDH 900. Esquistocitos en
el frotis de sangre periférica. Recibe nifedipino
vía oral sin respuesta. ¿Cuál es la conducta más
adecuada?
A. Completar la maduración pulmonar fetal
B. Terapia con antihipertensivos y sulfato de
magnesio
C. Manejo expectante con controles cada 48 h
D. Finalización inmediata de la gestación
EXAMEN NACIONAL DE MEDICINA 2021
wwwww.qxmedic.com
www.grupoqxmedic.com
165. Mujer de 45 años, desde hace 10 días presenta
disfagia para alimentos sólidos. Desde hace 48
horas fiebre de 39 °C, trismus y dificultad
respiratoria. Al examen: desviación medial del
paladar blando. Hemograma con 18000
leucocitos y 7% abastonados. TAC cervical:
colección de 30 ml en orofaringe. ¿Cuál es el
diagnóstico más probable?
A. Absceso dentario
B. Absceso periamigdaliano
C. Angina de Ludwig
D. Amigdalitis aguda
166. Varón de 45 años, víctima de accidente de
tránsito es llevado a la emergencia. Al examen:
lúcido, muy quejumbroso, equimosis a nivel de
la novena y décima costilla izquierda con
crepitación y dolor a la palpación. ECO FAST:
Líquido libre en cavidad aproximadamente 300
ml. ¿Cuál es el órgano que está probablemente
lesionado?
A. Cola de páncreas
B. Fondo gástrico
C. Ángulo esplénico del colon
D. Bazo
167. Mujer de 30 años se encuentra amamantando
a su bebe. ¿Qué respuesta hormonal se espera
en la madre?
A. Aumento de la secreción de oxitocina
B. Disminución de secreción de FSH
C. Disminución de la secreción de prolactina
D. Aumento de la secreción de vasopresina
168. Gestante de 37 semanas en tratamiento con
TARGA, con carga viral de 1500 copias/ml,
acude a Emergencia con dilatación de 1 cm,
altura de presentación -1, borramiento al 100%
y membranas ovulares íntegras. ¿Cuál es la
conducta más riesgosa para el recién nacido?
A. Parto por cesárea
B. Zidovudina endovenosa
C. Parto por vía vaginal
D. Continuar con TARGA
169. Mujer de 36 años acude a emergencia por
presentar desde hace 6 días, escalofríos,
sensación de alza térmica y malestar general,
disuria y tenesmo. Examen Físico: T 38.5° C,
dolor difuso en abdomen y pelvis. Examen de
Orina: Presencia de nitritos y cilindros
leucocitarios. ¿Cuál es el diagnóstico probable?
A. Pielonefritis
B. Cistitis
C. Vaginitis
D. Uretritis
170. Niña de 4 años presenta desde hace un mes
palidez, fatiga y dolor óseo en piernas. Al
examen: hepatoesplenomegalia. Laboratorio:
Hb 7 g/dl, plaquetas 20000 mm3, leucocitos
35000 mm3, linfoblastos 1%. ¿Cuál es la
conducta a seguir?
A. Biopsia hepática
B. Radiografía de extremidades
C. Ecografía abdominal
D. Aspirado de médula ósea
171. Mujer de 28 años acude por emergencia con
herida profunda en muslo izquierdo de 4 días
de evolución. Dolor intenso en muslo izquierdo
y alza térmica, siendo atendida en posta
médica. Examen: herida cruenta con flogosis,
vesículas de color marrón oscuro, secreción de
agua con aspecto de lavado de carne. ¿Cuál es
el diagnóstico más probable?
A. Seroma aplastronado
B. Absceso multilocular
C. Fasceitis necrotizante
D. Hematoma infectado
172. Puérpera inmediata de 27 años, sometida a
cesárea de emergencia presentando sangrado
de 500 ml en el acto quirúrgico. Tres horas
después hipotensión arterial de 60/40 mmHg,
FC 120 x' e incremento de la palidez. El tipo de
shock que estaría presentando es de
A. Continente (distributivo)
B. Obstructivo
C. Bomba (cardiogénico)
D. Contenido (hipovolémico)
EXAMEN NACIONAL DE MEDICINA 2021
wwwww.qxmedic.com
www.grupoqxmedic.com
173. Mujer de 60 años sin antecedentes de
enfermedad vascular previa, refiere dolor en
pie y región sural derecha. Al examen: ausencia
de pulso pedio y poplíteo con disminución de la
sensibilidad, frialdad en pie e incapacidad para
mover el grupo muscular afectado. ¿Cuál es el
diagnóstico probable?
A. Embolia arterial
B. Insuficiencia venosa profunda
C. Trombosis ateroesclerótica
D. Espasmo arterial
174. Mujer de 30 años, desde hace 5 meses tiene
molestias urinarias recurrentes. Examen de
orina muestra pH ácido, leucocitos 60 x campo.
Urocultivo negativo ¿Cuál es la sospecha
diagnostica?
A. Absceso perirrenal
B. Cistitis crónica
C. Pielonefritis crónica
D. Tuberculosis urogenital
175. Mujer de 25 años sexualmente activa con
amenorrea de 9 semanas, desde hace 24 horas
presenta dolor tipo cólico en hipogastrio y
sangrado escaso. Al examen: funciones vitales
estables, ansiosa con facies dolorosa, cérvix
blando, cerrado, de aspecto violáceo con
escaso sangrado ¿Cuál es el procedimiento a
realizar?
A. Indicar culdocentesis
B. Solicitar ecografía transvaginal
C. Solicitar B-hCG
D. Preparar para legrado uterino
176. Varón de 30 años hace 12 horas sufrió golpe en
la cabeza con pérdida de conocimiento, al
momento presenta cefalea global de
intensidad moderada, vómitos explosivos,
mareos y fotofobia. Fondo de ojo: edema de
papilas. Glasgow 10 y descendiendo. ¿Cuál
indicación es prioritaria?
A. Solución cristaloide isosmótica
B. Dimehidrinato sódico
C. Antiinflamatorios no esteroideos
D. Solución salina hipertónica
177. Lactante de 1 mes hospitalizado por presentar
paroxismo de tos con estridor y ahogo desde
hace una semana. Antecedente: hermano
mayor presentó episodio similar hace dos
semanas. ¿Cuál es el tratamiento antibiótico de
primera línea?
A. Cotrimoxazol
B. Amoxicilina
C. Azitromicina
D. Clindamicina
178. Varón de 76 años que, desde hace 1 año,
presenta polaquiuria, disuria y urgencia vesical,
hace dos meses vaciado incompleto de vejiga y
goteo postmiccional. Al tacto: próstata
aumentada de tamaño en forma difusa. PSA 5
ng/ml.
¿Cuál es el tratamiento inicial?
A. Agonistas de la GnRH
B. Resección transuretral de la próstata
C. Bloqueadores alfa adrenérgicos
D. Vaporización de la próstata con láser
179. Varón de 60 años, procedente de zona rural de
Puno, con un día de dolor abdominal intenso y
marcada distensión, asociados a vómitos
fecaloideos. RX de abdomen: signo de grano de
café. No antecedentes quirúrgicos. ¿Cuál es el
diagnóstico más probable?
A. Intususcepción intestinal
B. Perforación de víscera hueca
C. Diverticulitis aguda
D. Vólvulo de sigmoides
180. Varón de 67 años trabaja 15 años como obrero
en una fábrica de papel. Presenta disnea y tos
desde hace 1 año. Examen: PA 130/80 mmHg,
FR 26 x' acropaquias y crepitantes gruesos en
bases de ambos hemitórax. RX tórax: imágenes
reticulonodulares basales y bilaterales,
pulmones pequeños, SaO2 94% en reposo y
72% al esfuerzo. ¿Cuál es el diagnóstico más
probable?
A. Enfisema pulmonar
B. Enfermedad pulmonar intersticial difusa
C. Enfermedad pulmonar obstructiva crónica
D. Hipertensión pulmonar

Weitere ähnliche Inhalte

Was ist angesagt?

Orquiepididimitis
OrquiepididimitisOrquiepididimitis
OrquiepididimitisNoe2468
 
Sindrome Nefrótico y Nefrítico en Pediatría
Sindrome Nefrótico y Nefrítico en PediatríaSindrome Nefrótico y Nefrítico en Pediatría
Sindrome Nefrótico y Nefrítico en PediatríaAlonso Custodio
 
GUÍA 2018 COLANGITIS AGUDA
GUÍA 2018 COLANGITIS AGUDAGUÍA 2018 COLANGITIS AGUDA
GUÍA 2018 COLANGITIS AGUDAUci Grau
 
Sepsis severa y choque septico
Sepsis severa y choque septicoSepsis severa y choque septico
Sepsis severa y choque septicoCarlos Pech Lugo
 
GINECOLOGIA/OBSTETRICIA: Enfermedad inflamatoria pélvica
GINECOLOGIA/OBSTETRICIA: Enfermedad inflamatoria pélvica GINECOLOGIA/OBSTETRICIA: Enfermedad inflamatoria pélvica
GINECOLOGIA/OBSTETRICIA: Enfermedad inflamatoria pélvica Jihan Simon Hasbun
 
Síndrome Nefrótico en Pediatría
Síndrome Nefrótico en PediatríaSíndrome Nefrótico en Pediatría
Síndrome Nefrótico en PediatríaJuan Meléndez
 
Sindrome de Escroto Agudo
Sindrome de Escroto AgudoSindrome de Escroto Agudo
Sindrome de Escroto AgudoAlonso Custodio
 
ORQUIEPIDIDIMITIS, TORCION TESTICULAR Y CA TESTICULAR
ORQUIEPIDIDIMITIS, TORCION TESTICULAR Y CA TESTICULARORQUIEPIDIDIMITIS, TORCION TESTICULAR Y CA TESTICULAR
ORQUIEPIDIDIMITIS, TORCION TESTICULAR Y CA TESTICULARPharmed Solutions Institute
 
Abdomen Agudo
Abdomen AgudoAbdomen Agudo
Abdomen AgudoFAMEN
 
Epiglotitis pediátrica
Epiglotitis pediátricaEpiglotitis pediátrica
Epiglotitis pediátricaSara Leal
 
enfermedad inflamatoria pelvica
enfermedad inflamatoria pelvicaenfermedad inflamatoria pelvica
enfermedad inflamatoria pelvicalesteryahh
 
Unidad II enfermedad pelvica inflamatoria - Fernanda gea - unica - cirugia
Unidad II  enfermedad pelvica inflamatoria - Fernanda  gea - unica - cirugiaUnidad II  enfermedad pelvica inflamatoria - Fernanda  gea - unica - cirugia
Unidad II enfermedad pelvica inflamatoria - Fernanda gea - unica - cirugiaFernanda Pineda Gea
 

Was ist angesagt? (20)

Orquiepididimitis
OrquiepididimitisOrquiepididimitis
Orquiepididimitis
 
Examen de-rm-2013-parte-b
Examen de-rm-2013-parte-bExamen de-rm-2013-parte-b
Examen de-rm-2013-parte-b
 
Sindrome Nefrótico y Nefrítico en Pediatría
Sindrome Nefrótico y Nefrítico en PediatríaSindrome Nefrótico y Nefrítico en Pediatría
Sindrome Nefrótico y Nefrítico en Pediatría
 
GUÍA 2018 COLANGITIS AGUDA
GUÍA 2018 COLANGITIS AGUDAGUÍA 2018 COLANGITIS AGUDA
GUÍA 2018 COLANGITIS AGUDA
 
Hiperplasia prostatica.
Hiperplasia prostatica.Hiperplasia prostatica.
Hiperplasia prostatica.
 
Sepsis severa y choque septico
Sepsis severa y choque septicoSepsis severa y choque septico
Sepsis severa y choque septico
 
Preguntas pancreatitis enarm
Preguntas pancreatitis enarmPreguntas pancreatitis enarm
Preguntas pancreatitis enarm
 
GINECOLOGIA/OBSTETRICIA: Enfermedad inflamatoria pélvica
GINECOLOGIA/OBSTETRICIA: Enfermedad inflamatoria pélvica GINECOLOGIA/OBSTETRICIA: Enfermedad inflamatoria pélvica
GINECOLOGIA/OBSTETRICIA: Enfermedad inflamatoria pélvica
 
Torsion testicular
Torsion testicularTorsion testicular
Torsion testicular
 
Síndrome Nefrótico en Pediatría
Síndrome Nefrótico en PediatríaSíndrome Nefrótico en Pediatría
Síndrome Nefrótico en Pediatría
 
Sindrome de Escroto Agudo
Sindrome de Escroto AgudoSindrome de Escroto Agudo
Sindrome de Escroto Agudo
 
ORQUIEPIDIDIMITIS, TORCION TESTICULAR Y CA TESTICULAR
ORQUIEPIDIDIMITIS, TORCION TESTICULAR Y CA TESTICULARORQUIEPIDIDIMITIS, TORCION TESTICULAR Y CA TESTICULAR
ORQUIEPIDIDIMITIS, TORCION TESTICULAR Y CA TESTICULAR
 
Abdomen Agudo
Abdomen AgudoAbdomen Agudo
Abdomen Agudo
 
Guias Tokyo - Pancreatitis
Guias Tokyo - PancreatitisGuias Tokyo - Pancreatitis
Guias Tokyo - Pancreatitis
 
Epiglotitis pediátrica
Epiglotitis pediátricaEpiglotitis pediátrica
Epiglotitis pediátrica
 
Técnica quirúrgica de la Paracentesis.
Técnica quirúrgica de la Paracentesis.Técnica quirúrgica de la Paracentesis.
Técnica quirúrgica de la Paracentesis.
 
enfermedad inflamatoria pelvica
enfermedad inflamatoria pelvicaenfermedad inflamatoria pelvica
enfermedad inflamatoria pelvica
 
APENDICITIS AGUDA
APENDICITIS AGUDAAPENDICITIS AGUDA
APENDICITIS AGUDA
 
Apendicitis aguda
Apendicitis agudaApendicitis aguda
Apendicitis aguda
 
Unidad II enfermedad pelvica inflamatoria - Fernanda gea - unica - cirugia
Unidad II  enfermedad pelvica inflamatoria - Fernanda  gea - unica - cirugiaUnidad II  enfermedad pelvica inflamatoria - Fernanda  gea - unica - cirugia
Unidad II enfermedad pelvica inflamatoria - Fernanda gea - unica - cirugia
 

Ähnlich wie ENAM 2021 - 20 DE MARZO.pdf

Ähnlich wie ENAM 2021 - 20 DE MARZO.pdf (20)

Enam 2021.pdf
Enam 2021.pdfEnam 2021.pdf
Enam 2021.pdf
 
Multiple choice
Multiple choiceMultiple choice
Multiple choice
 
68137138 enam-2010
68137138 enam-201068137138 enam-2010
68137138 enam-2010
 
Enam exam-2017concurso
Enam exam-2017concursoEnam exam-2017concurso
Enam exam-2017concurso
 
examen_enam25042021.pdf
examen_enam25042021.pdfexamen_enam25042021.pdf
examen_enam25042021.pdf
 
PREGUNTAS DEL EXAMEN NACIONAL DE MEDICINA 17/03/2024
PREGUNTAS DEL EXAMEN NACIONAL DE MEDICINA 17/03/2024PREGUNTAS DEL EXAMEN NACIONAL DE MEDICINA 17/03/2024
PREGUNTAS DEL EXAMEN NACIONAL DE MEDICINA 17/03/2024
 
preguntas de ginecología bancqueo de preguntas
preguntas de ginecología bancqueo de preguntaspreguntas de ginecología bancqueo de preguntas
preguntas de ginecología bancqueo de preguntas
 
Parte 1 de 3 essalud 2017
Parte 1 de 3 essalud 2017Parte 1 de 3 essalud 2017
Parte 1 de 3 essalud 2017
 
2015
20152015
2015
 
Examen rmn 2019.docx(2)
Examen   rmn 2019.docx(2)Examen   rmn 2019.docx(2)
Examen rmn 2019.docx(2)
 
segundo parcial laboratorio jose luis aguirre
segundo parcial laboratorio jose luis aguirresegundo parcial laboratorio jose luis aguirre
segundo parcial laboratorio jose luis aguirre
 
30788343 banco-de-preguntas-pediatria
30788343 banco-de-preguntas-pediatria30788343 banco-de-preguntas-pediatria
30788343 banco-de-preguntas-pediatria
 
Caso clinico para entregar
Caso clinico para entregarCaso clinico para entregar
Caso clinico para entregar
 
Ped 8
Ped 8Ped 8
Ped 8
 
BANCO DE PREGUNTAS 2DA SEM CLASE 1a.pptx
BANCO DE PREGUNTAS 2DA SEM CLASE 1a.pptxBANCO DE PREGUNTAS 2DA SEM CLASE 1a.pptx
BANCO DE PREGUNTAS 2DA SEM CLASE 1a.pptx
 
Exun 2014 a
Exun 2014  aExun 2014  a
Exun 2014 a
 
Exun 2014 a
Exun 2014  aExun 2014  a
Exun 2014 a
 
Exun 2015 a
Exun 2015   aExun 2015   a
Exun 2015 a
 
Caso martes adria
Caso martes adriaCaso martes adria
Caso martes adria
 
CPP- Casos clínicos TP 4 y 5.pptx
CPP- Casos clínicos TP 4 y 5.pptxCPP- Casos clínicos TP 4 y 5.pptx
CPP- Casos clínicos TP 4 y 5.pptx
 

Mehr von Edward leyva

edward - caso.pptx
edward - caso.pptxedward - caso.pptx
edward - caso.pptxEdward leyva
 
Embarazo Ectopico Internado.pptx
Embarazo Ectopico Internado.pptxEmbarazo Ectopico Internado.pptx
Embarazo Ectopico Internado.pptxEdward leyva
 
INCOMPATIBILIDAD ABO - RH - LEYVA BALTAZAR HANS EDWARD.pptx
INCOMPATIBILIDAD ABO - RH - LEYVA BALTAZAR HANS EDWARD.pptxINCOMPATIBILIDAD ABO - RH - LEYVA BALTAZAR HANS EDWARD.pptx
INCOMPATIBILIDAD ABO - RH - LEYVA BALTAZAR HANS EDWARD.pptxEdward leyva
 
EXPOSICION ASMA HOY.pptx
EXPOSICION ASMA HOY.pptxEXPOSICION ASMA HOY.pptx
EXPOSICION ASMA HOY.pptxEdward leyva
 
EXPOSICION ASMA5 HOY.pptx
EXPOSICION ASMA5 HOY.pptxEXPOSICION ASMA5 HOY.pptx
EXPOSICION ASMA5 HOY.pptxEdward leyva
 
2. Funciones Estadísticas.pdf
2. Funciones Estadísticas.pdf2. Funciones Estadísticas.pdf
2. Funciones Estadísticas.pdfEdward leyva
 
CANCER DE PANCREAS.pdf
CANCER DE PANCREAS.pdfCANCER DE PANCREAS.pdf
CANCER DE PANCREAS.pdfEdward leyva
 
CANCER DE PANCREAS (1) (2).pdf
CANCER DE PANCREAS (1) (2).pdfCANCER DE PANCREAS (1) (2).pdf
CANCER DE PANCREAS (1) (2).pdfEdward leyva
 
Tema B - Simulacro 270322.pdf
Tema B - Simulacro 270322.pdfTema B - Simulacro 270322.pdf
Tema B - Simulacro 270322.pdfEdward leyva
 
Tema A - Simulacro 270322.pdf
Tema A - Simulacro 270322.pdfTema A - Simulacro 270322.pdf
Tema A - Simulacro 270322.pdfEdward leyva
 
Realidad nacional ECO REGIONES
Realidad nacional ECO REGIONES Realidad nacional ECO REGIONES
Realidad nacional ECO REGIONES Edward leyva
 
UPSJB Realidad nacional
UPSJB Realidad nacional UPSJB Realidad nacional
UPSJB Realidad nacional Edward leyva
 

Mehr von Edward leyva (12)

edward - caso.pptx
edward - caso.pptxedward - caso.pptx
edward - caso.pptx
 
Embarazo Ectopico Internado.pptx
Embarazo Ectopico Internado.pptxEmbarazo Ectopico Internado.pptx
Embarazo Ectopico Internado.pptx
 
INCOMPATIBILIDAD ABO - RH - LEYVA BALTAZAR HANS EDWARD.pptx
INCOMPATIBILIDAD ABO - RH - LEYVA BALTAZAR HANS EDWARD.pptxINCOMPATIBILIDAD ABO - RH - LEYVA BALTAZAR HANS EDWARD.pptx
INCOMPATIBILIDAD ABO - RH - LEYVA BALTAZAR HANS EDWARD.pptx
 
EXPOSICION ASMA HOY.pptx
EXPOSICION ASMA HOY.pptxEXPOSICION ASMA HOY.pptx
EXPOSICION ASMA HOY.pptx
 
EXPOSICION ASMA5 HOY.pptx
EXPOSICION ASMA5 HOY.pptxEXPOSICION ASMA5 HOY.pptx
EXPOSICION ASMA5 HOY.pptx
 
2. Funciones Estadísticas.pdf
2. Funciones Estadísticas.pdf2. Funciones Estadísticas.pdf
2. Funciones Estadísticas.pdf
 
CANCER DE PANCREAS.pdf
CANCER DE PANCREAS.pdfCANCER DE PANCREAS.pdf
CANCER DE PANCREAS.pdf
 
CANCER DE PANCREAS (1) (2).pdf
CANCER DE PANCREAS (1) (2).pdfCANCER DE PANCREAS (1) (2).pdf
CANCER DE PANCREAS (1) (2).pdf
 
Tema B - Simulacro 270322.pdf
Tema B - Simulacro 270322.pdfTema B - Simulacro 270322.pdf
Tema B - Simulacro 270322.pdf
 
Tema A - Simulacro 270322.pdf
Tema A - Simulacro 270322.pdfTema A - Simulacro 270322.pdf
Tema A - Simulacro 270322.pdf
 
Realidad nacional ECO REGIONES
Realidad nacional ECO REGIONES Realidad nacional ECO REGIONES
Realidad nacional ECO REGIONES
 
UPSJB Realidad nacional
UPSJB Realidad nacional UPSJB Realidad nacional
UPSJB Realidad nacional
 

Kürzlich hochgeladen

RAIZ CUADRADA Y CUBICA PARA NIÑOS DE PRIMARIA
RAIZ CUADRADA Y CUBICA PARA NIÑOS DE PRIMARIARAIZ CUADRADA Y CUBICA PARA NIÑOS DE PRIMARIA
RAIZ CUADRADA Y CUBICA PARA NIÑOS DE PRIMARIACarlos Campaña Montenegro
 
EXPANSIÓN ECONÓMICA DE OCCIDENTE LEÓN.pptx
EXPANSIÓN ECONÓMICA DE OCCIDENTE LEÓN.pptxEXPANSIÓN ECONÓMICA DE OCCIDENTE LEÓN.pptx
EXPANSIÓN ECONÓMICA DE OCCIDENTE LEÓN.pptxPryhaSalam
 
SINTAXIS DE LA ORACIÓN SIMPLE 2023-2024.pptx
SINTAXIS DE LA ORACIÓN SIMPLE 2023-2024.pptxSINTAXIS DE LA ORACIÓN SIMPLE 2023-2024.pptx
SINTAXIS DE LA ORACIÓN SIMPLE 2023-2024.pptxlclcarmen
 
UNIDAD DPCC. 2DO. DE SECUNDARIA DEL 2024
UNIDAD DPCC. 2DO. DE  SECUNDARIA DEL 2024UNIDAD DPCC. 2DO. DE  SECUNDARIA DEL 2024
UNIDAD DPCC. 2DO. DE SECUNDARIA DEL 2024AndreRiva2
 
DE LAS OLIMPIADAS GRIEGAS A LAS DEL MUNDO MODERNO.ppt
DE LAS OLIMPIADAS GRIEGAS A LAS DEL MUNDO MODERNO.pptDE LAS OLIMPIADAS GRIEGAS A LAS DEL MUNDO MODERNO.ppt
DE LAS OLIMPIADAS GRIEGAS A LAS DEL MUNDO MODERNO.pptELENA GALLARDO PAÚLS
 
Clasificaciones, modalidades y tendencias de investigación educativa.
Clasificaciones, modalidades y tendencias de investigación educativa.Clasificaciones, modalidades y tendencias de investigación educativa.
Clasificaciones, modalidades y tendencias de investigación educativa.José Luis Palma
 
DECÁGOLO DEL GENERAL ELOY ALFARO DELGADO
DECÁGOLO DEL GENERAL ELOY ALFARO DELGADODECÁGOLO DEL GENERAL ELOY ALFARO DELGADO
DECÁGOLO DEL GENERAL ELOY ALFARO DELGADOJosé Luis Palma
 
Heinsohn Privacidad y Ciberseguridad para el sector educativo
Heinsohn Privacidad y Ciberseguridad para el sector educativoHeinsohn Privacidad y Ciberseguridad para el sector educativo
Heinsohn Privacidad y Ciberseguridad para el sector educativoFundación YOD YOD
 
Sesión de aprendizaje Planifica Textos argumentativo.docx
Sesión de aprendizaje Planifica Textos argumentativo.docxSesión de aprendizaje Planifica Textos argumentativo.docx
Sesión de aprendizaje Planifica Textos argumentativo.docxMaritzaRetamozoVera
 
Caja de herramientas de inteligencia artificial para la academia y la investi...
Caja de herramientas de inteligencia artificial para la academia y la investi...Caja de herramientas de inteligencia artificial para la academia y la investi...
Caja de herramientas de inteligencia artificial para la academia y la investi...Lourdes Feria
 
ACUERDO MINISTERIAL 078-ORGANISMOS ESCOLARES..pptx
ACUERDO MINISTERIAL 078-ORGANISMOS ESCOLARES..pptxACUERDO MINISTERIAL 078-ORGANISMOS ESCOLARES..pptx
ACUERDO MINISTERIAL 078-ORGANISMOS ESCOLARES..pptxzulyvero07
 
La empresa sostenible: Principales Características, Barreras para su Avance y...
La empresa sostenible: Principales Características, Barreras para su Avance y...La empresa sostenible: Principales Características, Barreras para su Avance y...
La empresa sostenible: Principales Características, Barreras para su Avance y...JonathanCovena1
 
OLIMPIADA DEL CONOCIMIENTO INFANTIL 2024.pptx
OLIMPIADA DEL CONOCIMIENTO INFANTIL 2024.pptxOLIMPIADA DEL CONOCIMIENTO INFANTIL 2024.pptx
OLIMPIADA DEL CONOCIMIENTO INFANTIL 2024.pptxjosetrinidadchavez
 
NARRACIONES SOBRE LA VIDA DEL GENERAL ELOY ALFARO
NARRACIONES SOBRE LA VIDA DEL GENERAL ELOY ALFARONARRACIONES SOBRE LA VIDA DEL GENERAL ELOY ALFARO
NARRACIONES SOBRE LA VIDA DEL GENERAL ELOY ALFAROJosé Luis Palma
 
cortes de luz abril 2024 en la provincia de tungurahua
cortes de luz abril 2024 en la provincia de tungurahuacortes de luz abril 2024 en la provincia de tungurahua
cortes de luz abril 2024 en la provincia de tungurahuaDANNYISAACCARVAJALGA
 
La triple Naturaleza del Hombre estudio.
La triple Naturaleza del Hombre estudio.La triple Naturaleza del Hombre estudio.
La triple Naturaleza del Hombre estudio.amayarogel
 
MAYO 1 PROYECTO día de la madre el amor más grande
MAYO 1 PROYECTO día de la madre el amor más grandeMAYO 1 PROYECTO día de la madre el amor más grande
MAYO 1 PROYECTO día de la madre el amor más grandeMarjorie Burga
 
Identificación de componentes Hardware del PC
Identificación de componentes Hardware del PCIdentificación de componentes Hardware del PC
Identificación de componentes Hardware del PCCesarFernandez937857
 

Kürzlich hochgeladen (20)

RAIZ CUADRADA Y CUBICA PARA NIÑOS DE PRIMARIA
RAIZ CUADRADA Y CUBICA PARA NIÑOS DE PRIMARIARAIZ CUADRADA Y CUBICA PARA NIÑOS DE PRIMARIA
RAIZ CUADRADA Y CUBICA PARA NIÑOS DE PRIMARIA
 
EXPANSIÓN ECONÓMICA DE OCCIDENTE LEÓN.pptx
EXPANSIÓN ECONÓMICA DE OCCIDENTE LEÓN.pptxEXPANSIÓN ECONÓMICA DE OCCIDENTE LEÓN.pptx
EXPANSIÓN ECONÓMICA DE OCCIDENTE LEÓN.pptx
 
SINTAXIS DE LA ORACIÓN SIMPLE 2023-2024.pptx
SINTAXIS DE LA ORACIÓN SIMPLE 2023-2024.pptxSINTAXIS DE LA ORACIÓN SIMPLE 2023-2024.pptx
SINTAXIS DE LA ORACIÓN SIMPLE 2023-2024.pptx
 
UNIDAD DPCC. 2DO. DE SECUNDARIA DEL 2024
UNIDAD DPCC. 2DO. DE  SECUNDARIA DEL 2024UNIDAD DPCC. 2DO. DE  SECUNDARIA DEL 2024
UNIDAD DPCC. 2DO. DE SECUNDARIA DEL 2024
 
DE LAS OLIMPIADAS GRIEGAS A LAS DEL MUNDO MODERNO.ppt
DE LAS OLIMPIADAS GRIEGAS A LAS DEL MUNDO MODERNO.pptDE LAS OLIMPIADAS GRIEGAS A LAS DEL MUNDO MODERNO.ppt
DE LAS OLIMPIADAS GRIEGAS A LAS DEL MUNDO MODERNO.ppt
 
Clasificaciones, modalidades y tendencias de investigación educativa.
Clasificaciones, modalidades y tendencias de investigación educativa.Clasificaciones, modalidades y tendencias de investigación educativa.
Clasificaciones, modalidades y tendencias de investigación educativa.
 
DECÁGOLO DEL GENERAL ELOY ALFARO DELGADO
DECÁGOLO DEL GENERAL ELOY ALFARO DELGADODECÁGOLO DEL GENERAL ELOY ALFARO DELGADO
DECÁGOLO DEL GENERAL ELOY ALFARO DELGADO
 
Heinsohn Privacidad y Ciberseguridad para el sector educativo
Heinsohn Privacidad y Ciberseguridad para el sector educativoHeinsohn Privacidad y Ciberseguridad para el sector educativo
Heinsohn Privacidad y Ciberseguridad para el sector educativo
 
Presentacion Metodología de Enseñanza Multigrado
Presentacion Metodología de Enseñanza MultigradoPresentacion Metodología de Enseñanza Multigrado
Presentacion Metodología de Enseñanza Multigrado
 
Sesión de aprendizaje Planifica Textos argumentativo.docx
Sesión de aprendizaje Planifica Textos argumentativo.docxSesión de aprendizaje Planifica Textos argumentativo.docx
Sesión de aprendizaje Planifica Textos argumentativo.docx
 
Caja de herramientas de inteligencia artificial para la academia y la investi...
Caja de herramientas de inteligencia artificial para la academia y la investi...Caja de herramientas de inteligencia artificial para la academia y la investi...
Caja de herramientas de inteligencia artificial para la academia y la investi...
 
ACUERDO MINISTERIAL 078-ORGANISMOS ESCOLARES..pptx
ACUERDO MINISTERIAL 078-ORGANISMOS ESCOLARES..pptxACUERDO MINISTERIAL 078-ORGANISMOS ESCOLARES..pptx
ACUERDO MINISTERIAL 078-ORGANISMOS ESCOLARES..pptx
 
La empresa sostenible: Principales Características, Barreras para su Avance y...
La empresa sostenible: Principales Características, Barreras para su Avance y...La empresa sostenible: Principales Características, Barreras para su Avance y...
La empresa sostenible: Principales Características, Barreras para su Avance y...
 
OLIMPIADA DEL CONOCIMIENTO INFANTIL 2024.pptx
OLIMPIADA DEL CONOCIMIENTO INFANTIL 2024.pptxOLIMPIADA DEL CONOCIMIENTO INFANTIL 2024.pptx
OLIMPIADA DEL CONOCIMIENTO INFANTIL 2024.pptx
 
NARRACIONES SOBRE LA VIDA DEL GENERAL ELOY ALFARO
NARRACIONES SOBRE LA VIDA DEL GENERAL ELOY ALFARONARRACIONES SOBRE LA VIDA DEL GENERAL ELOY ALFARO
NARRACIONES SOBRE LA VIDA DEL GENERAL ELOY ALFARO
 
cortes de luz abril 2024 en la provincia de tungurahua
cortes de luz abril 2024 en la provincia de tungurahuacortes de luz abril 2024 en la provincia de tungurahua
cortes de luz abril 2024 en la provincia de tungurahua
 
La triple Naturaleza del Hombre estudio.
La triple Naturaleza del Hombre estudio.La triple Naturaleza del Hombre estudio.
La triple Naturaleza del Hombre estudio.
 
Medición del Movimiento Online 2024.pptx
Medición del Movimiento Online 2024.pptxMedición del Movimiento Online 2024.pptx
Medición del Movimiento Online 2024.pptx
 
MAYO 1 PROYECTO día de la madre el amor más grande
MAYO 1 PROYECTO día de la madre el amor más grandeMAYO 1 PROYECTO día de la madre el amor más grande
MAYO 1 PROYECTO día de la madre el amor más grande
 
Identificación de componentes Hardware del PC
Identificación de componentes Hardware del PCIdentificación de componentes Hardware del PC
Identificación de componentes Hardware del PC
 

ENAM 2021 - 20 DE MARZO.pdf

  • 1. EXAMEN NACIONAL DE MEDICINA 2021 wwwww.qxmedic.com www.grupoqxmedic.com 1. Se evidencia el incremento de pacientes afebriles que acuden a la consulta y el servicio de salud ambiental informa la presencia de Aedes aegypti en el ámbito territorial del centro de salud. En este caso, ¿cuál será la acción prioritaria de los equipos extramurales de salud? A. Notificación internacional inmediata B. Control de contactos y fuentes de infección C. Aplicación de vacuna contra el dengue D. Tratamiento antibiótico domiciliario de casos 2. Lactante de 9 meses presenta hace 4 días fiebre, deposiciones con moco y sangre, pujo y tenesmo. Al examen: signos de deshidratación moderada, somnoliento, abdomen distendido, doloroso, RHA aumentados. Se envía muestra para coprocultivo. Se inicia hidratación parenteral y se administra........... A. Amoxicilina B. Cloranfenicol C. Ceftriazona D. Ampicilina 3. Lactante varón de 6 meses, alimentado con sustituto de leche materna. Inicia hace dos días con fiebre, rinorrea y tos. Al tercer día se añade dificultad respiratoria por lo que es traído a consulta. Niega antecedentes patológicos. Al examen: taquipneico con politirajes y sibilancias espiratorias. ¿Cuál es el diagnóstico probable? A. Resfrío común B. Cuerpo extraño C. Bronquitis D. Bronquiolitis 4. Varón de 33 años, acude al servicio de urología por presentar disminución de volumen testicular. Antecedente: hace un año sometido a cirugía por traumatismo a nivel de pelvis. ¿Qué complicación quirúrgica explica este cuadro? A. Ablación del cuerpo del epidídimo B. Ligadura de arteria testicular C. Sección del conducto deferente D. Desgarro de la arteria funicular 5. Paciente de 45 años, procedente de distrito endémico del dengue, acude al puesto de salud por presentar fiebre persistente, dolor retro ocular, dolor abdominal intenso y vómitos. ¿Cuál es la clasificación inicial del caso? A. Brote epidémico por dengue B. Sospechoso de dengue clásico C. Sindrome de choque por dengue D. Probable dengue con signos de alarma 6. Mujer de 21 años, con perfil lipídico: colesterol total 390 mg/dl, triglicéridos 125 mg/dl, HDL 45 mg/dl y LDL 320 mg/dl. Al examen: xantomas en talón de Aquiles y extensores de las manos. Padre con colesterol y LDL muy aumentados. Hermana con colesterol de 400 mg/dl. Madre y hermano sin alteraciones. ¿Cuál es el trastorno más probable? A. Hipercolesterolemia poligénica B. Hiperlipidemia familiar combinada C. Hipercolesterolemia familiar D. Hipertrigliceridemia familiar 7. Mujer de 68 años, consulta por sensación bulto y sequedad en zona vaginal. Al examen: se evidencia distopia genital grado I según POPQ. ¿Cuál es el manejo más adecuado? A. Estrógenos locales B. Colpoperineoplastía C. Uso de pesario D. Observación y control en 1 año 8. Mujer de 22 años, consulta por flujo vaginal amarillento, espumoso, con mal olor y prurito vaginal. Al examen: se encuentra colpitis en fresa, ¿cuál es el agente causal? A. Gardnerella vaginalis B. Trichomonas vaginalis C. Herpes simple D. Candida albicans 9. Segundigesta de 12 semanas, asintomática, con sobrepeso y lupus eritematoso sistémico no activo. Antecedente: preeclampsia con criterios de severidad y parto pretérmino en última gestación. ¿Qué medida sería la más efectiva como profilaxis para la preeclampsia? A. Dieta baja en sal B. Ácido acetil salicílico a dosis bajas C. Suplemento de vitamina D D. Enoxaparina
  • 2. EXAMEN NACIONAL DE MEDICINA 2021 wwwww.qxmedic.com www.grupoqxmedic.com 10. Varón de 28 años, llevado a emergencia por los bomberos, quienes refieren haberlo encontrado "tirado en la calle". Al examen: FC 62 x', FR 12 x', SaO2 90%, T 35.6 °C, PA 100/60 mmHg, soporoso, pupilas fotoreactivas. Se le administra flumazenilo 0.2 mg EV y el paciente despierta. ¿Qué sustancia química ingirió el paciente? A. Alcohol B. Antidepresivos C. Morfina D. Benzodiacepinas 11. Varón de 45 años sufre accidente de tránsito. Presenta dolor, intensidad 9/10 e impotencia funcional. Se diagnostica luxación posterior de cadera, la reducción es realizada a las 72 horas. ¿Cuál es complicación ósea que se podría presentar a mediano o largo plazo? A. Anquilosis ósea B. Necrosis avascular C. Fractura del acetábulo D. Lesión del nervio ciático 12. Niño de 3 años, hace 7 días inicia con edema en párpados y tobillos. Laboratorio: colesterol 280 mg/dl, albúmina 1.5 mg/dl, proteinuria 40 mg/m2/hora, creatinina sérica 1.1 mg/dl, hematíes en orina 3 a 4 x campo, PA sistólica percentil 50, C3 normal. ¿Cuál es el diagnóstico probable? A. Sindrome nefrótico B. Insuficiencia renal C. Sindrome nefrítico D. Glomerulonefritis 13. Paciente de 23 años, con diagnóstico de asma bronquial desde su niñez, viene por crisis que se inició hace 3 horas. Al examen PA 110/70 mmHg, FC 130 x', FR 32 x', despierto con politirajes y silencio en la auscultación ambos hemitórax. De todo lo expuesto, ¿qué indica una peor evolución? A. Politirajes B. Polipnea C. Silencio auscultatorio D. Duración de la crisis 14. Varón de 60 años, consulta por aparición de tumoraciones cervicales y axilares indoloras. Al examen: PA 110/70 mmHg, FC 89 x', T 38.9 °C, se palpan adenopatías cervicales y axilares indoloras, de consistencia elástica. Resto sin alteraciones. ¿Cuál es el procedimiento más recomendable para el diagnóstico? A. Extirpación de ganglio cervical y estudio anatomopatológico B. Tomografía de tórax, abdomen y pelvis. C. Biopsia de médula ósea por aspiración D. Endoscopía digestiva y fibrobroncoscopía pulmonar 15. Varón de 44 años, se le realiza endoscopía digestiva alta, evidenciándose zona de excoriación de mucosa a nivel de curvatura menor, cerca del antro y del píloro. ¿Cuál es la causa básica que origina este hallazgo? A. Aumento de producción de moco por las glándulas gástricas pilóricas profundas B. Desequilibrio entre la secreción gástrica y el grado de protección de la mucosa C. Deficiente acción neutralizante biliar frente al ácido gástrico D. Aumento de la secreción de bicarbonato de sodio 16. Se desea estudiar el impacto de elaborar preguntas de opción múltiple con 3 alternativas de respuesta versus 5 alternativas. ¿Qué tipo de diseño de investigación debe aplicarse? A. Estudio experimental B. Casos y controles C. Estudio descriptivo D. Estudio de cohortes 17. Paciente de 66 años admitido en UCI por trauma craneoencefálico grave que progresa a diagnóstico establecido de muerte cerebral. En junta médica se decide suspender el soporte ventilatorio. ¿Qué valor ético se estaría vulnerando? A. Solidaridad B. Respeto a la persona C. Ninguno D. Justicia
  • 3. EXAMEN NACIONAL DE MEDICINA 2021 wwwww.qxmedic.com www.grupoqxmedic.com 18. En una zona urbana de extrema pobreza hay una alta concentración de casos positivos de tuberculosis pulmonar. El centro de salud informa que tiene asegurados los medicamentos para el tratamiento completo de los pacientes, sin embargo, algunos pacientes presentan serias limitaciones para cumplir el tratamiento supervisado. ¿Cuál es la principal razón para esta falta de adherencia? A. Insuficiente conocimiento sobre la enfermedad B. Horario laboral C. Ubicación de la vivienda D. Disponibilidad de personal 19. Escolar de 6 años, presenta malestar general, anorexia, dolor abdominal periumbilical de 12 horas de evolución, acompañado de vómitos sin contenido alimentario, no diarreas. Al examen: dolor a la palpación en fosa iliaca derecha, ¿cuál es el diagnóstico probable? A. Apendicitis aguda B. Invaginación intestinal C. Gastroenterocolitis D. Diverticulitis 20. En su SERUMS usted evalúa a una mujer de 19 años que suele acudir con su pareja por presentar dolor pélvico crónico e infecciones pélvicas a repetición. Siempre abandona tratamiento. Al examen: se muestra ansiosa, incómoda, evita contacto visual y mira constantemente a su pareja. ¿Cuál sería su principal sospecha diagnóstica? A. Sindrome depresivo B. Endometriosis moderada C. Violencia doméstica D. Sindrome conversivo 21. Lactante de 8 meses, traído a consulta por presentar desde hace 5 días fiebre, rinorrea y tos esporádica. Hoy el niño presenta llanto constante, rechaza la alimentación y se golpea compulsivamente con ambas manos las orejas. ¿Qué examen confirmaría el diagnóstico? A. Otoscopía B. Visualización de la faringe C. Auscultación de los pulmones D. Búsqueda de reflejo rojo 22. En una análisis de heces de rutina se encuentran huevos de Strongyloides stercoralis. ¿Cuál de las siguientes opciones sería el tratamiento de primera elección? A. Centro de Salud Mental Comunitario B. Mebendazol C. Ivermectina D. Albendazol 23. Puérpera inmediata con diabetes gestacional. Usted desea valorar en el posparto su estatus metabólico para manejo. ¿En qué momento le indica dosaje de glucosa? A. A la semana B. De 1 a 3 días C. A las cuatro semanas D. A los 15 días 24. Mujer de 47 años, obesa e hipertensa. Acude a consultorio de ginecología por presentar sangrado uterino luego de 4 años de amenorrea. Cérvix uterino normal. PAP negativo. Ecografía transvaginal: endometrio 6mm. ¿Cuál es la prescripción adecuada? A. Histerectomía B. Biopsia de endometrio C. Observación D. Tomografía contrastada abdominopélvica 25. Varón de 22 años, trasladado por el SAMU por sufrir asalto con arma blanca. Al examen: herida profunda sangrante en la nuca. No puede movilizar el hemicuerpo derecho y la sensibilidad está afectada en el lado contralateral. ¿Qué estudio es recomendable para el diagnóstico? A. Ecografía de partes blandas B. Gammagrafía cervical C. TAC cervical contrastada D. Resonancia magnética cervical
  • 4. EXAMEN NACIONAL DE MEDICINA 2021 wwwww.qxmedic.com www.grupoqxmedic.com 26. Primigesta de 32 semanas por FUR, acude a Emergencia por dolor abdominal tipo cólico y pérdida de moco. Movimientos fetales normales. Al examen: contracciones uterinas de 30 segundos de duración cada 4 minutos, intensidad ++. Latidos fetales 158 x'. No pérdida de líquido a la maniobra de Valsalva. Tacto vaginal: cérvix blando, dilatación 1 cm, membranas íntegras. ¿Cuál es la conducta a seguir? A. Hospitalización e inducción del parto B. Reposo en casa C. Maduración pulmonar y tocólisis D. Término de gestación por cesárea 27. Primigesta de 40 semanas en trabajo de parto. Luego de 2 horas no presenta modificaciones en la dilatación cervical. Dinámica uterina 2/10, intensidad ++/+++, duración 40 segundos. Tacto vaginal: dilatación de 6 cm, altura de presentación -3, membranas ovulares rotas con líquido claro. ¿Cuál es la conducta a seguir? A. Colocar a la paciente en decúbito lateral izquierdo B. Cesárea C. Acentuación del trabajo de parto D. Reevaluación clínica de la pelvis y del feto 28. Niño de 8 meses que presenta vómitos y diarrea, se diagnostica deshidratación moderada y se decide dar hidratación oral. El peso actual es de 8 kg. Se prescribe sales de hidratación oral a razón de por 4 horas A. A voluntad cada hora B. 200 ml cada hora C. 100 ml cada hora D. 50 ml cada hora 29. Varón de 68 años con EPOC, acude a Emergencia por presentar súbitamente dolor torácico y disnea que no responde a oxigenoterapia ni a broncodilatadores. Al examen: 110/60 mmHg, FC 110 x', FR 30 x', SaO2 86%, hiperresonancia a la percusión de hemitórax derecho. ¿Cuál es el diagnóstico probable? A. Neumotórax agudo B. Embolismo pulmonar C. Exacerbación del EPOC D. Neumonía lobar 30. Lactante de 1 año con rechazo al alimento desde hace 1 mes. Al examen: desnutrido, pálido, cefalohetoma parietal izquierdo, piernas con lesiones hipocrómicas curvadas de 0.8 y 10 mm de longitud, manchas equimóticas redondeadas en muslos y zona escrotal. ¿Cuál es el diagnóstico probable? A. Sepsis fulminante B. Maltrato infantil C. Enfermedad de Kawasaki D. Déficit de antitrombina 31. Paciente usuaria de DIU acude por retraso menstrual de tres días. Se le solicita un test de embarazo en orina y sale positivo. Al examen: se aprecian los hilos del dispositivo. ¿Cuál es la conducta a seguir? A. Repetir test de embarazo B. Solicitar una ecografía transvaginal C. Solicitar un β-hCG cuantitativa seriada D. Retirar el dispositivo intrauterino 32. Recién nacido de 3500 g con hipotermia, débil succión, hipoactividad, signos de dificultad respiratoria, polipneico y tiraje subcostal. Leucocitos 3000 x mm3. Relación abastonados/neutrófilos de 0.3. ¿Cuál es el tratamiento de primera elección luego de la toma de cultivos? A. Ceftriaxona + amikacina B. Ampicilina + amikacina C. Vancomicina + cefotaxima D. Ampicilina + cefotaxima 33. Varón de 50 años, fumador pesado, desde hace 1 año viene presentando episodios de dolor opresivo retroesternal que duran 5 minutos aproximadamente, desencadenados luego de practicar deportes y que mejora con nitroglicerina. ¿Cuál es el diagnóstico más probable? A. Angina de esfuerzo B. Infarto agudo de miocardio C. Estenosis mitral D. Angina estable
  • 5. EXAMEN NACIONAL DE MEDICINA 2021 wwwww.qxmedic.com www.grupoqxmedic.com 34. Taxista de 42 años, desde hace 1 año lesiones en piel y desde 6 meses hinchazón y dolor articular. Al examen: lesiones cutáneas, eritematosas descamativas y pruriginosas en codos. Laboratorio: ANA, factor reumatoide y serología de lúes negativos; VSG 50 mm/h. RX mano: erosiones periarticulares en interfalángicas distales de 2do y 3er dedo de mano derecha. ¿Cuál es el diagnóstico probable? A. Artritis reumatoide B. Artritis psoriásica C. Síndrome de Reiter D. Artropatía amiloide 35. Mujer de 50 años que presenta sensación persistente de tristeza y desesperanza, dificultad para concentrarse e ideas suicidas. Refiere que últimamente escucha voces y ve personas que sus familiares ni oyen ni ven. ¿Cuál es el diagnóstico más probable? A. Trastorno afectivo estacional B. Depresión mayor C. Distimia D. Depresión psicótica 36. La enfermera comunica al médico jefe que va a devolver las vacunas para el Covid-19 porque la población no solicita su aplicación. El médico encuentra que el 90% de la población ha recibido la primera y segunda dosis, y solo el 30% la tercera dosis. Convoca a todo el personal para formar brigadas de vacunación a domicilio. La campaña resulta exitosa y se logra la cobertura. ¿Qué habilidad de gestión ha aplicado en este nivel el médico jefe? A. Comunicación B. Profesionalismo C. Adaptación al cambio D. Liderazgo 37. Al servicio de urgencia del centro de salud acude un número de personas por mordedura de perro. Refieren que son perros callejeros y no es posible su captura, ante eso el médico jefe decide realizar una coordinación intersectorial. ¿A qué institución deberá convocar prioritariamente? A. Municipalidad B. Fiscalía de prevención C. Policía Nacional D. Dirección de educación 38. Gestante de 38 semanas con IMC 24, se muestra preocupada porque en todo el embarazo ha ganado 15 kg. Según la mayoría de los autores, la ganancia ponderal para ella es: A. Muy alta B. Alta C. Baja D. Adecuada 39. Varón de 65 años, diabético desde hace 20 años, ingresa por emergencia con trastorno de conciencia y respiración de Kussmaul. Laboratorio: glucosa 400 mg/dl, pH 7.23, pCO2 30 mmHg, HCO3 18 mEq/L. Orina: presencia de betahidroxibutirato. ¿Qué cambios fisiológicos esperaría encontrar? A. Disminución de la excreción renal de bicarbonato B. Disminución de la excreción de amonio en la orina C. Aumento de la excreción pulmonar de amonio D. Disminución de la eliminación pulmonar de CO2 40. Varón de 39 años con diagnóstico COVID-19, acude a urgencia con su hijo de 13 años quien presenta fiebre, cefalea y dolor de garganta. El padre refiere que hace dos semanas en su centro laboral se diagnosticaron varios casos de COVID-19. ¿A qué tipo de epidemia corresponde? A. Problemas de conectividad en la educación virtual B. Exposición continuada C. Fuente común D. Mixta 41. Un médico prescribe antibióticos de marca a un niño "porque los genéricos no son tan efectivos". La madre del paciente hará un esfuerzo para adquirirlos a pesar de que carece de recursos económicos. ¿Qué principio bioético se ha vulnerado? A. Justicia B. Autonomía C. Beneficencia D. Maleficencia
  • 6. EXAMEN NACIONAL DE MEDICINA 2021 wwwww.qxmedic.com www.grupoqxmedic.com 42. Madre refiere que su hijo de 2 años presenta hace 24 horas dolor abdominal tipo cólico, se acompaña de náuseas y vómitos. Abdomen: se palpa masa alargada dolorosa en epigastrio, RHA aumentados con timbre metálico. RX abdomen simple: niveles hidroaéreos de asas intestinales delgadas. TAC de abdomen: signo de la "salchicha", líquido libre en cavidad aproximadamente 200 cm3. ¿Cuál es la conducta terapéutica inmediata a seguir? A. Enema con aire B. Colonoscopia C. Colon con enema D. Laparotomía exploratoria 43. Mujer de 30 años, luego un expulsivo prolongado da a luz a un bebé de 4100 g, alumbramiento normal. En el puerperio inmediato presenta sangrado vaginal abundante. Al examen: PA 50/20 mmHg, FC 120 x', sopor y palidez marcada. Al nivel de la cicatriz umbilical se pala útero blando. ¿Cuál es el diagnóstico correcto? A. Laceración cervical B. Tromboastenia de Glanzmann C. Acretismo placentario D. Atonía uterina 44. Niño de 2 años que, en ambiente poco iluminados, se choca con las cosas, siente molestia con la luz. En la evaluación presenta xerosis conjuntival y manchas de Bitot. ¿Qué deficiencia vitamínica se relaciona con estos hallazgos? A. K B. C C. E D. A 45. Mujer de 75 años con hipertensión arterial y anticoagulación por fibrilación auricular. Llevada a Emergencia por trastorno de conciencia. Al examen: PA 208/108 mmHg, FC 108 x', FR 16 x', SaO2 96%, T 36.8 °C, afasia y hemiparesia derecha. Laboratorio: INR 1,6. ¿Cuál es la primera acción a realizar? A. Reducir la presión arterial sistólica < 185 mmHg B. Trombólisis para revascularizar C. Optimizar anticoagulación D. Administrar vitamina K 46. Varón de 40 años es traído por familiares a Emergencia e ingresa a Trauma Shock, por dificultad respiratoria progresiva. Usted observa que el paciente deja de respirar, no responde al llamado y no hay pulso carotídeo. ¿Cuál es el primer paso de actuación? A. Interconsulta a cardiología B. Desfibrilación inmediata C. Intubación orotraqueal D. Compresión cardiaca 47. Madre con su hijo de 5 meses acude a control de crecimiento y desarrollo, solicita orientación para inicio de alimentación complementaria. Si se requiere prevenir la anemia por deficiencia de hierro. ¿Qué indicación se le dará? A. Priorizar papillas en base a frutas y sopas B. Alimentar con sucedáneos de leche materna C. Incluir diariamente alimentos de origen animal D. Dar término a la lactancia materna 48. Mujer de 45 años luego de larga caminata presenta dolor con aumento de volumen de la rodilla izquierda. Artrocentesis: leucocitosis de 12000 x mm3. ¿Cómo clasificaría a este líquido? A. Séptico B. Normal C. Inflamatorio D. Hemorrágico 49. Varón de 23 años sufre atricción por caída de pared sobre ambos miembros inferiores, luego de unas horas es llevado de emergencia por presentar oligoanuria. ¿Qué examen requiere para establecer el diagnóstico? A. Hematocrito seriado B. Sedimento urinario C. CPK total D. Sodio sérico 50. Mujer de 34 años, consulta por amenorrea de 8 meses, pérdida de cabello y sequedad vaginal. IMC 24. Laboratorio: FSH 44 mUI/ml, estradiol 20 pg/ml, prolactina 10 ug/dl. ¿Cuál es el diagnóstico más probable? A. Falla ovárica prematura B. Síndrome de ovario poliquístico C. Hiperprolactinemia D. Síndrome de Cushing
  • 7. EXAMEN NACIONAL DE MEDICINA 2021 wwwww.qxmedic.com www.grupoqxmedic.com 51. Mujer de 74 años refiere pérdida brusca de la visión del ojo derecho con dolor intenso. Antecedente de asma bronquial y de alergia a las sulfonamidas. La lámpara de hendidura revela edema del epitelio corneal, ángulo esclerocorneal cerrado y cámara anterior poco profunda. ¿Cuál es el tratamiento a indicar? A. Manitol B. Dorzolamida C. Timolol D. Latanoprost 52. Gestante de 28 semanas con pielonefritis aguda en resolución. ¿Cuál es el manejo para prevenir la reinfección en esta gestación? A. Amoxicilina 500 mg diario B. Urocultivo semanal C. Examen completo de orina semanal D. Nitrofurantoina 100 mg diario 53. Al inicio de la pandemia COVID-19 no se contaba con vacuna, por lo cual, para disminuir el riesgo de contagio, solo se tenía evidencia de protección mediante el uso de mascarilla, lavado de manos, distanciamiento físico. ¿A través de qué intervención se logró que la población cumpla con estas medidas? A. Uso de medio masivos de comunicación B. Educación a domicilio C. Escenificación teatral en parques D. Charlas en centros laborales 54. Varón de 68 años, en terapia con hidroclorotiazida. Desde hace 15 días presenta debilidad muscular, incluso se ha caído en dos ocasiones. Al examen: presenta debilidad generalizada con hiporreflexia. ¿Cuál es el procedimiento inicial? A. TAC cerebral B. Electromiografía C. Electroencefalografía D. Electrocardiografía 55. Un tamizaje de violencia familiar realizado por profesionales del consultorio de salud de la mujer reveló que la incidencia es alta, decidiéndose que las víctimas recibieran atención integral especializada refiriéndolas a: A. Centro de Salud Mental Comunitario B. Ministerio de la Mujer C. Hospital de tercer nivel D. Instituto Nacional de Salud Mental 56. Mujer de 62 años con antecedente de hipertensión arterial. Presenta bruscamente hemiplejia derecha con afasia no afluente que revierte íntegramente luego de 15 minutos. Tomografía cerebral normal. ¿Cuál es el diagnóstico? A. Hemorragia subaracnoidea B. Hemorragia intraparenquimal C. Embolia cerebral D. Ataque isquémico transitorio 57. Varón de 28 años ingresa a Emergencia por atropello vehicular. Al examen: PA 100/60 mmHg, FC 110 x', FR 28 x', SaO2 92% con una FiO2 40%, herida contuso cortante en región parietal derecha, abdomen distendido, RHA presentes y fractura fémur derecho. Al estímulo doloroso abre los ojos, emite sonidos incomprensibles y vuelve a caer en sopor. ¿Cuál es la primera acción a seguir? A. Intubación orotraqueal B. Suturar herida de la cabeza C. Colocar vía central D. Radiografía de cráneo 58. Mujer de 50 años consulta por dolor e hinchazón de pierna izquierda de una semana de evolución. Al examen: Aumento de volumen de pierna izquierda. No signos inflamatorios. Signo de Homans positivo. Dímero D: negativo. ¿Cuál es el diagnóstico más probable? A. Trombosis venosa profunda B. Paniculitis septal C. Infección de partes blandas D. Linfangitis crónica 59. Varón de 62 años fumador crónico de 2 paquetes diarios de cigarrillos. Desde hace 3 meses, tos persistente con expectoración hemoptoica y baja de peso 5 kg. BK negativo por 3 veces. RX de tórax: masa suprahiliar izquierda de 5 cm de diámetro. ¿Qué examen confirmaría el diagnóstico? A. Broncofibroscopía con biopsia B. TAC pulmonar con contraste C. Biopsia por punción transtorácica D. Resonancia magnética pulmonar
  • 8. EXAMEN NACIONAL DE MEDICINA 2021 wwwww.qxmedic.com www.grupoqxmedic.com 60. Lactante mujer de 11 meses nacida por cesárea debido a presentación podálica y macrosomía. Al examen: asimetría de pliegues en glúteos y signo de Galeazzi positivo. ¿Cuál es el diagnóstico probable? A. Artritis de cadera B. Agenesia sacra C. Artrosis de cadera D. Displasia de cadera 61. Mujer de 74 años, sufre de hipermetropía, desde hace 24 horas viene presentando dolor en el ojo izquierdo con pérdida visual paulatina, hiperemia ocular con opacidad de córnea y midriasis. A la palpación aumento de la tonicidad del ojo. ¿Cuál es el probable diagnóstico? A. Parálisis del III par craneal B. Glaucoma crónico C. Uveítis D. Glaucoma agudo 62. Mujer de 43 años llegó a Lima para trabajar hace 3 meses y desde entonces presenta insomnio, cambios de humor e irritabilidad. Habla poco, es muy desconfiada, en el trabajo se desenvuelve lo mejor que puede y se alegra cuando escucha noticias de su terruño o de sus familiares. ¿Qué trastorno se evidencia en este caso? A. Disociativo B. Somatización C. Adaptación D. Ansioso-depresivo 63. Neonato a término en segundo día de vida con Trisomía 21. Se evidencia intolerancia oral, vómitos biliosos, abdomen moderadamente distendido, timpánico, no signos de irritación peritoneal. ¿Cuál sería el diagnóstico más probable? A. Atresia duodenal B. Atresia esofágica C. Estenosis hipertrófica de píloro D. Atresia de colon distal 64. Varón de 50 años que se automedicó durante 7 días con altas dosis de aspirina, azitromicina e ivermectina por sospecha de infección por SARS-CoV-2. Requirió endoscopia digestiva alta por presentar abundante sangrado. ¿Cuál es la causa básica de esta complicación? A. Inmunosupresión B. Agotamiento de vitamina K C. Lisis de eritrocitos D. Inhibición de la ciclooxigenasa 65. Varón de 50 años acude por dolor abdominal intenso en epigastrio con irradiación a región lumbar que se inició hace 7 días, hace 2 días se agrega náuseas e hiporexia. Antecedente de alcoholismo crónico. ¿Qué examen debe solicitar para confirmar diagnóstico? A. Fosfatasa alcalina B. Amilasa C. Lipasa D. Transaminasas 66. Un organismo no gubernamental financiará intervenciones para la disminución de las incidencias de embarazo en adolescentes con la condición de que estas se reduzcan por lo menos en 10% al año. ¿Cual es la intervención inicial que el equipo de gestión debe asegurar para minimizara incertidumbre y lograr el resultado esperado? A. Liderazgo del gobierno local B. Calidad de información C. Contrato de obstetras D. Adecuación cultural del parto 67. Mujer de 50 años, con Diabetes Mellitus tipo 2 de 15 años de evolución, con mal control metabólico. Refiere desde hace 1 año dolor y parestesias en MMII. Al examen: pérdida de sensibilidad al dolor en MMII. ¿Cuál es el medicamento más apropiado para este problema? A. Tiamina B. Gabapentina C. Tramadol D. Propanolol
  • 9. EXAMEN NACIONAL DE MEDICINA 2021 wwwww.qxmedic.com www.grupoqxmedic.com 68. Niña de 2 años, inicia hace 5 días con fiebre, cuadro catarral intenso con rinorrea, conjuntivitis, fotofobia y tos seca. Al examen: manchas en la mucosa yugal, erupción exantemática maculopapular morbiliforme rojo vinoso y confluente generalizada en todo el cuerpo. ¿Qué antecedente es importante conocer para un diagnóstico probable? A. Esquema de vacunación antisarampión B. Si tuvo contacto con varicela C. Si tiene alergia a medicamento o alimento D. Si tuvo contacto con garrapatas 69. Albañil de 48 años, portador de tumoración inguino escrotal izquierda, reductible y asintomática. Abdomen: Tumoración blanda de 5 cm de diámetro, no dolorosa a la palpación. Maniobra de Valsalva (+), maniobra de Coley (-). ¡Cuál es el probable diagnóstico? A. Hernia crural B. Eventración abdominal C. Hernia inguinal directa D. Hernia inguinal indirecta 70. Segundigesta de 28 semanas, presenta contracciones uterinas desde hace 12 horas. Primer hijo pretérmino nacido por cesárea. Al examen: dinámica uterina 2 en 10 minutos, intensidad ++ y duración de 30 segundos. Tacto vaginal: dilatación 4 cm, borramiento 80%. ¿Cuáles el diagnóstico? A. Pródromos de trabajo de parto B. Labor de parto pretérmino C. Amenaza de labor de parto pretérmino D. Incompetencia cervical 71. Adolescente de 15 años, por primera vez ayuda en la cosecha de uvas, presenta dolor abdominal tipo retortijón, vómitos, diarrea y fasciculaciones musculares. ¿Cuál es la sospecha diagnóstica? A. Intoxicación alimentaria B. Hipoglicemia C. Neurosis conversiva D. Intoxicación por insecticidas 72. El equipo de gestión de un centro de salud I-4 encuentra que en los últimos 5 años se ha incrementado la población adulta mayor, de los cuales 60% son de sexo femenino, 95% están en situación de pobreza extrema y 90% cuentan con seguro integral de salud. ¿Cuál fase del análisis de la situación de salud esta gestionando? A. Priorización de problemas B. Identificación de actores C. Definición de líneas de acción D. Análisis del entorno 73. Recién nacido masculino, antecedente obstétrico de polihidramnios. Presenta vómitos biliosos. Al examen: abdomen distendido, RHA incrementados. RX abdomen: Imagen de doble burbuja. ¿Cuál es el diagnóstico probable? A. Atresia duodenal B. Malrotación intestinal C. Estenosis hipertrófica del píloro D. Hernia diafragmática 74. Varón de 65 años, hace 5 meses fiebre intermitente, cefalea y artralgias. Al examen: dolor a la presión en la región temporal bilateral, claudicación de la mandíbula y amaurosis bilateral. Laboratorio: Hb 8.5 g/dl, VSG 150 mm/h. ¿Cuál es la conducta terapéutica a seguir? A. Ciclofosfamida B. Metrotexate C. Ácido acetilsalicílico D. Corticoesteroides 75. Mujer de 50 años, presenta cólicos biliares a repetición desde hace 2 años. Hace 24 horas intenso dolor abdominal en epigastrio con náuseas y vómitos. Distención abdominal. Amilasa 3000 UI/d, Lipasa 600 UI/d, Bilirrubina totales 3 mg/dl, BD 2.5 mg/dl, BI 0,5 mg/dl. ¿Cuál examen de imagen solicitaría? A. Tomografía abdominal B. Colangiografía retrógrada endoscópica C. Ecografía abdominal D. Gammagrafía de hígado
  • 10. EXAMEN NACIONAL DE MEDICINA 2021 wwwww.qxmedic.com www.grupoqxmedic.com 76. Varón de 28 años, desde hace 3 días, fiebre, cefalea, mialgias intensas. Antecedente de haber visitado Iquitos hace 9 días. Al tomar la presión arterial, aparece un gran número de petequias en el antebrazo. ¿Cuál es el diagnóstico más probable? A. Malaria B. Fiebre amarilla C. Dengue D. Leptospirosis 77. Adolescente de 12 años presenta cefalea desde hace 1 mes. Se asocia tos a predominio nocturno y rinorrea intermitente. Al examen: secreción purulenta en la pared posterior de la faringe. ¿Cuál es el diagnóstico a plantear? A. Sinusitis subaguda B. Rinitis alérgica C. Rinofaringitis crónica D. Rinosinusitis aguda 78. Niño de 6 años, desde hace 3 días presenta tos productiva. Dolor retroesternal al toser y al respirar profundamente. Al examen: roncantes diseminados en ambos hemitórax. ¿Cuál es el diagnóstico probable? A. Traqueobronquitis B. Bronquiolitis aguda C. Bronquitis aguda D. Neumonía intersticial 79. Varón de 65 años, desde hace 8 semanas refiere llenura precoz, disminución de peso y distención abdominal. Al examen: adelgazado, pálido, se palpa adenopatía supraclavicular izquierda de consistencia pétrea y abdomen globuloso con signo de oleada positivo. Laboratorio: anemia microcítica e hipocrómica. ¿Qué examen es necesario para proponer un diagnóstico? A. Biopsia ganglionar B. Paracentesis exploratoria C. Endoscopia digestiva alta D. Colonoscopía virtual 80. Mujer de 34 años, consulta por baja de peso de aproximadamente 6 kilos en un mes. Refiere incremento de apetito y sed, intolerancia al calor y sudoración excesiva. Al examen: PA 120/50 mmHg, FC 120 x', FR 16 x', T 37,5 C piel húmeda y caliente. ¿Cuál es el diagnóstico probable? A. Adenopatía hipofisiario B. Hipertiroidismo C. Feocromocitoma D. Diabetes tipo 2 81. Varón de 13 años, desde hace tres días presenta dolor en rodilla derecha que le impide caminar y fiebre de 38,5 C. Al examen: rodilla derecha dolorosa, aumentada de volumen y con signos inflamatorios. Hemograma con leucocitos de 25000 x mm³ y 10% de abastonados. ¿Cuál es el probable germen responsable de la infección? A. Staphylococcus aureus B. Escherichia coli C. Corynebacterium pyogenes D. Haemophilus influenzae 82. Chofer de ambulancia colisiona contra un poste y es conducido a Emergencia con fractura expuesta de fémur derecho. Al examen: PA 100/60 mmHg, herida en muslo con exposición de plano muscular. RM: fémur con trazo de fractura diafisaria no desplazada. ¿Cuál es el manejo inicial que indicaría? A. Colgajo muscular B. Reducción cruenta C. Desbridamiento D. Injerto cutáneo 83. Mujer de 64 años VIH (+), desde hace 3 días dolor urente en la región lateral de hemitórax izquierdo, seguido de eritema y vesículas de tamaño variable a lo largo del dermatoma. Test de Tzanck: cambios citoplasmáticos típicos y presencia e células gigantes multinucleadas. ¿Cuál es el diagnóstico más probable? A. Herpes zóster B. Dermatitis de contacto C. Pioderma gangrenoso D. Dermatosis neutrofílica
  • 11. EXAMEN NACIONAL DE MEDICINA 2021 wwwww.qxmedic.com www.grupoqxmedic.com 84. Gestante de 33 semanas de embarazo sin control prenatal, acude por sangrado vaginal, no refiere otras molestias. Cesareada 1 vez hace 3 años. Al examen: PA 100/60 mmHg, FC 86 x', AU 30 cm, feto en LCI, LCF 130 x' presentación flotante a la cuarta maniobra de Leopold. No se constataron contracciones uterinas. ¿Qué procedimiento deberíamos evitar en esta paciente? A. Ecografía obstétrica B. Tacto vaginal C. Especuloscopía D. Ecografía transvaginal 85. Puérpera mediata de 3 días por cesárea. Inicia con fiebre 38.5 C en 2 horas separadas. Al examen: se palpa útero doloroso a 2 cm de la cicatriz umbilical. A la especuloscopía: secreción purulenta. ¿Cuál es la conducta a seguir? A. Explorar herida operatoria B. Solicitar ecografía transvaginal C. Preparación para legrado puerperal D. Iniciar antibioticoterapia 86. Varón 83 años acude por sangrado nasal severo. Examen: funciones vitales estables, palidez leve de piel y mucosas. Se observa sangrado en regular cantidad por fosa nasal derecha y por boca. ¿Cuál es el procedimiento de elección? A. Taponamiento posterior B. Taponamiento Anterior C. Cauterización del vestíbulo nasal D. Compresión nasal externa 87. Mujer de 36 años con antecedentes de litiasis urinaria de 5 años. Desde hace 30 días dolor lumbar derecha y fiebre continua. Al examen: PPL derecha +++. Laboratorio: leucocitosis 14500/mm³ sin desviación izquierda, Hb 9,5 g/dl y VSG 92 mm/h. Sedimento urinario: leucocitos 50 x campo y la citología urinaria muestra abundantes macrófagos con aspecto espumoso. ¿Cuál es el diagnóstico más probable? A. Adenocarcinoma renal B. Pielonefritis xantogranulomatosa C. Absceso perinéfrico D. Pielonefritis enfisematosa 88. Mujer de 30 años, puérpera de 2 semanas. Consulta por fiebre. Al examen: dolor, enrojecimiento e induración de piel en mama derecha. ¿Cuál es la actitud a seguir? A. Antibiótico y seguimiento clínico B. Mamografía de mama C. Biopsia con aguja gruesa D. Drenaje y control en 2 semanas 89. Mujer de 20 años, acude al servicio de emergencia porque hace 2 horas se quemó con agua hervida. Al examen: piel con ampollas y flictenas que abarcan toda la extremidad superior derecha. El compromiso de superficie corporal corresponde al _____ %. A. 9 B. 27 C. 4.5 D. 18 90. Tercigesta de 39 semanas por FUR en labor de parto, con dolor abdominal súbito y sangrado vaginal. Cesareada anterior con periodo intergenésico corto. Al examen: se palpan partes fetales en abdomen. Tacto vaginal: dilatación 3, estación -3, membranas rotas con líquido sanguinolento. Latidos fetales 100 x'. ¿Cuál es el diagnóstico? A. Desprendimiento prematuro de placenta B. Rotura uterina C. Rotura de vasa previa D. Placenta previa 91. Un centro de salud reporta aumento en la incidencia de tuberculosis pulmonar BK positivo. El análisis de la situación de salud del ámbito muestra hacinamiento y aumento del desempleo ¿Cuál de los siguientes aspectos es el de mayor influencia en este problema? A. Limitada búsqueda de sintomáticos B. No uso de mascarilla C. Determinantes sociales D. Multidrogoresistencia
  • 12. EXAMEN NACIONAL DE MEDICINA 2021 wwwww.qxmedic.com www.grupoqxmedic.com 92. Varón de 25 años consulta por infertilidad. Los exámenes auxiliares evidencian un problema de maduración de los espermatozoides. Los exámenes de la esposa resultaron normales. La explicación a esta situación se debe a disfunción del/de la…… A. cuerpo del epidídimo B. conducto eyaculador C. vesícula seminal D. conducto deferente 93. Mujer de 22 años acude porque súbitamente presenta dolor lumbar izquierdo irradiado hacia región inguinal del mismo lado que no calma con analgésicos. Al examen PPL izquierdo +++ Examen de orina: hematíes 20 a 50 x campo, cilindros eritrocitarios y cristales de oxalato. ¿Cuál es el probable diagnóstico? A. Cistitis aguda B. Litiasis vesical C. Litiasis renal D. Uretritis 94. Lactante de 5 meses de edad, durante su primer control en un establecimiento de salud 1-2 se detectó una ganancia de peso mensual menor de 500 g. No fue llevado a sus citas posteriores. ¿Qué acción se debe priorizar? A. Denunciar a los padres B. Visita domiciliaria C. Insistir en programación mensual D. Referencia a tercer nivel de atención 95. Mujer de 16 años con historia de menstruación prolongada hasta 15 días. Al examen: se detecta palidez, Hb 8 g/dl; pruebas para hemofilia deficiencia importante del factor von Willebrand. ¿Cuál es la terapia más adecuada? A. Transfusión de plaquetas B. Plasma fresco congelado C. Plasma residual D. Transfusión de sangre total 96. Un médico serumista , jefe de un establecimiento de salud 1-4, requiere presentar el plan operativo anual ¿Con cuál de los siguientes procesos debe iniciar la elaboración del plan? A. Formulación de metas y objetivos B. Determinación de los problemas sanitarios C. Determinación de actividades D. Asignación de recursos 97. Multípara de 41 semanas acude por contracciones uterinas 3/10 minutos, 40 segundos de duración, intensidad +++/+++, con placenta de inserción baja por ecografía y que sangra en escasa cantidad. Tacto vaginal: dilatación 7, altura de presentación -1, membranas integras, LCF 148 x'. ¿Cuál sería el manejo adecuado? A. Preparar a la paciente para operación cesárea B. Poner vía endovenosa, romper bolsas y esperar el parto vaginal C. Hospitalización y conducta expectante D. Acentuación del parto con oxitocina 98. Varón de 65 años, traído a Emergencia por sincope de esfuerzo. Al examen: PA 130/80 mmHg, despierto, pulso irregular. EKG (ver imagen). ¿Cuál es el diagnóstico más probable? A. Insuficiencia aórtica B. Miocardiopatía hipertrófica C. Taponamiento cardiaco D. Bloqueo A-V completo 99. Mujer de 54 años, reiteradamente acude a Emergencia por cefalea pulsátil, náuseas y vómitos En la anamnesis: habita en ambiente cerrado y refiere usar estufa de gas. Al examen: PA 110/60 mmHg. FC 120 X’ , FR 26 x’, SaO2 95% a aire ambiental. Desorientada, confusa e intensa debilidad muscular. ¿Cuál es la terapia más adecuada? A. Óxido nitroso en inhalación B. Intubación endotraqueal C. Oxígeno al 100% D. Ventilación a alto flujo a 30 It/min
  • 13. EXAMEN NACIONAL DE MEDICINA 2021 wwwww.qxmedic.com www.grupoqxmedic.com 100. Niño de 10 años, desde hace 4 días presenta fiebre dolor abdominal vómitos y coluria Antecedente: consume alimentos en el quiosco escolar. Al examen: ictericia y hepatomegalia. Laboratorio: TGO 740, TGP 1020. Tiempo de protrombina normal. ¿Cuál es el manejo apropiado? A. Gammaglobulina especifica y sintomáticos B. Hepatoprotectores y fitomenadiona C. Hidratación y sintomáticos D. Administrar Interferón gamma a dosis terapéutica 101. Adolescente de 13 años, hospitalizada por deshidratación y desnutrición con IMC 15. La madre refiere que con frecuencia su hija se provoca vómitos. ¿Qué trastorno del medio interno espera encontrar? A. Acidosis metabólica B. Acidosis hipoclorémica C. Alcalosis hiperclorémica D. Alcalosis hipoclorémica 102. Varón de 24 años acude por fiebre y artralgias. Al examen: acné severo con diferentes lesiones, incluso cicatriciales, en cara y tórax. Hemograma: leucocitosis. ¿Cuál es el tipo de acné mas probable? A. Pustuloso B. Fulminante C. Nodular D. Vulgar 103. Varón de 45 años, obeso acude a emergencia por presentar, dolor abdominal al epigástrico irradiado a la espalda, vómitos biliosos incoercibles. Al examen FC: 120 x’, PA 80/60 mmHg, soporosos, dolor abdominal difuso, ictericia. Triglicéridos elevados, Amilasa normal. ¿Cuál es la conducta inicial a seguir? A. Hidratación EV agresiva B. Alimentación enteral C. CPRE urgente D. Antibiótico profilaxis 104. Primogénito de un mes de edad, llevado a Emergencia por haber presentado vómitos en proyectil, no bilioso. Al examen: deshidratación moderada, se palpa tumoración pequeña en epigastrio. RX abdomen simple: burbuja de gas en el estómago. ¿Cuál es el diagnóstico? A. Estenosis yeyunal B. Estenosis duodenal C. Hipertrofia de píloro D. Gastrosquisis 105. Varón de 70 años con antecedente de alcoholismo, sufre caída desde una altura de 5 metros ingresa a Trauma Shock con Glasgow 10. Tomografía cerebral con Imagen refringente en forma de medialuna, con bordes poco definidos. ¿Cuál es el tratamiento indicado? A. Administrar vitamina K. B. Administrar coagulantes C. Craneotomía con drenaje D. Derivación ventrículo atrial 106. Lactante de 2 años desde hace 5 días presenta tos exigente Hace un día se agrega fiebre y respiración rápida. Al examen FC 172 X’ , FR 50 X’ T° 40 °C Sao2 96% tirajes subcostal e intercostal moderados. Hemitórax derecho amplexación y murmullo vesicular disminuidos y crepitantes diseminados. La principal sospecha es neumonía … A. adquirida en la comunidad B. atípica C. complicada con efusión D. complicada con empiema 107. Mujer de 38 años, asmática, acude a emergencia por inicio súbito de disnea y tos. Al examen obesidad centrípeta FC 120 x’ FR 30x’ SaO2 90 % confusa responde con frases cortas. A la auscultación sibilantes diseminados en ambos hemitórax ¿Cuál es la conducta inmediata a seguir? A. Intubación orotraqueal B. Administración de corticoides C. Nebulización con B2 agonistas D. Infusión de sulfato de magnesio
  • 14. EXAMEN NACIONAL DE MEDICINA 2021 wwwww.qxmedic.com www.grupoqxmedic.com 108. Neonato a término hospitalizado con diagnóstico de periodo de adaptación prolongado, con indicación de suplemento de oxígeno mínimo continuo. Hemoglucotest 45 mg/dl ¿Cual es la conducta a seguir? A. Iniciar bolo dextrosa al 10% B. Iniciar tratamiento antibiótico C. Reevaluación del hemoglucotest D. Restringir vía oral 109. Paciente agredido con un objeto penetrante a nivel del quinto espacio intercostal línea media clavicular izquierda. Funciones vitales PA 90/60 mmHg, FC 110 X’ , PVC 16 mmHg . Al examen ingurgitación yugular izquierda y ruidos cardiacos apagados ¿cuál es la conducta inmediata a seguir? A. Pericardiocentesis B. Fluidoterapia con cristaloides C. Pase a cuidados intensivos D. Drenaje pleural izquierdo 110. Recién nacido a término, a las 3 horas de vida presenta cianosis perioral y ungueal. La administración de oxigeno no mejora el color de la piel ni la saturación. ¿Cuál es el diagnostico probable? A. Persistencia del conducto arterioso B. Comunicación interventricular C. Estenosis pulmonar D. Comunicación interauricular 111. Varón de 70 años hipertenso antiguo, evaluado en Centro de salud por inicio reciente de disnea cansancio y fatiga. Al examen FR 40 x’ , FC 120x’, pa 130/70 mmHg, no tolera el decúbito, ingurgitación yugular + + +, crepitantes y sibilancias en ambos campos pulmonares. La conducta a seguir es: A. Diuréticos vía endovenosa B. Inhibidores de la enzima convertidora de la angiotensina C. Referencia inmediata al establecimiento de mayor complejidad D. Bloqueadores Beta adrenérgicos 112. ¿Qué principio bioético se vulnera cuando se brinda asistencia mecánica ventilatoria a un paciente adulto , que previamente había documentado su deseo de no ser intubado ni conectado a un ventilador mecánico A. No maleficencia B. Justicia C. Beneficencia D. Autonomía 113. Mujer de 50 años se resbala y al caerse el peso de su cuerpo es amortiguado por su extremidad superior izquierda. Al examen deformación y aumento de volumen de brazo izquierdo, pulsos presentes. RX: fractura del tercio medio de la diáfisis del húmero ¿Qué manifestación clínica se debe presentar? A. Mano péndula B. Parestesia hipotenar C. Mano en garra D. Llenado capilar prolongado 114. Se desea hacer una investigación sobre el imparto de la cuarentena por COVID-19 en el estado nutricional de estudiantes de medicina de una universidad pública. El marco conceptual de la investigación debe incluir prioritariamente. A. problemas de conectividad en la educación virtual B. Niveles de estrés educativo en universitarios C. Criterios de estilos de vida saludable en adultos jóvenes D. Secuela por confinamiento 115. Lactante de 1 año traído por haber presentado rigidez con relajación de esfínteres que duró unos minutos. No presenta antecedentes patológicos. Al examen Fc 120 X’ t38 °C, despierto hipoactivo, nos signos de meníngeos, tono muscular y reflejos normales ¿Cuál es el manejo inmediato? A. Bajar la temperatura B. Interconsulta a neurología C. Administración anticonvulsivantes D. Realizar electroencefalograma
  • 15. EXAMEN NACIONAL DE MEDICINA 2021 wwwww.qxmedic.com www.grupoqxmedic.com 116. Mujer de 22 años desde hace 1 hora presenta sensación de falta de aire, sudoración, palpitaciones , dolor torácico opresivo y temblor generalizado que la hacen sentir “ que se va a morir”. El cuadro cede espontáneamente. Refiere que el episodio suele repetirse de forma esporádica ante situaciones de estrés ¿Cuál es el diagnóstico más probable? A. Crisis de pánico B. Depresión mayor C. Trastorno obsesivo D. Síndrome de abstinencia 117. Varón de 69 años con diagnostico de pancreatitis crónica y alcoholismo crónico, desde hace 6 meses muestra aumento en la frecuencia de deposiciones las cuales son de consistencia semisólida, de aspecto pálido, brillantes y untuosas. ¿Cuál es el diagnostico más probable? A. Insuficiencia pancreática exocrina B. Síndrome Zollinger-Ellison C. Ampuloma D. Gastritis atrófica 118. Agricultor de 40 años , acude a centro de salud por presentar hace 4 días fiebre, cefalea artralgias y mialgias especialmente en pantorrillas . Refiere exposición a canales de regadío durante el último mes ¿Cuál es la clasificación del caso? A. Probable leptospirosis B. Sospecha de peste C. Confirmado de Dengue D. Contacto de fiebre amarilla 119. Médico infectólogo de 45 años, desde hace 3 meses se ha aislado de sus compañeros de trabajo. Refiere que ha inventado una vacuna contra el coronavirus y que por eso le darán el Premio Nobel. Además, expresa que le quiere robar su invento y que quieren matarlo. Cree que le han puesto micrófonos y cámaras en su departamento. Al examen: desaseado y ansioso. ¿Cuál es su diagnóstico probable? A. Sindrome de Burnout B. Psicosis depresiva C. Esquizofrenia paranoide D. Trastorno Bipolar 120. Mujer de 30 años, desde hace 3 meses presenta episodios de palpitaciones, cefalea e hiperhidrosis. Al examen: PA 180/120 mmHg, FC 100 x', piel pálida, fría y sudorosa; ruidos cardiacos de buena intensidad. Laboratorio: elevación de catecolaminas y metanefrinas urinarias. ¿Cuál es el diagnóstico más probable? A. Hipertensión renovascular B. Feocromocitoma C. Hiperaldosteronismo D. Hipertensión arterial esencial 121. Lactante de 10 meses, es traído a Emergencia por presentar temperatura de 39.5 C. La madre refiere que presentó retroversión ocular, movimientos generalizados con relajación de esfínteres y pérdida de la conciencia durante 30 minutos. Luego despertó, lactó durante 5 minutos y quedó dormido. ¿Cuál es el diagnóstico probable? A. Convulsión febril simple B. Epilepsia parcial continua C. Estatus epiléptico febril D. Convulsión febril compleja 122. Primigesta de 33 semanas por FUR, acude al centro de salud donde usted está haciendo su SERUMS. Refiere pérdida de líquido hace 8 horas que mojó hasta las rodillas. Signos vitales normales. No se evidencian contracciones uterinas. Al tacto vaginal el cuello uterino no está dilatado, pero sí reblandecido y el líquido amniótico es claro. Latidos fetales 154 x'. ¿Cuál es el manejo más adecuado? A. Tocolisis, hidratación y referencia a hospital de mayor complejidad B. Maduración pulmonar, profilaxis antibiótica y referencia a hospital de mayor complejidad C. Neuroprotección, maduración pulmonar y reevaluación en 4 horas D. Profilaxis antibiótica y evolución espontánea de trabajo de parto
  • 16. EXAMEN NACIONAL DE MEDICINA 2021 wwwww.qxmedic.com www.grupoqxmedic.com 123. Mujer de 61 años con EPOC acude a emergencia por presentar mayor disnea, incremento de tos y de esputo RX de tórax: signos de enfisema sin opacidades focales. Laboratorio: PaO2 62mmHg. ¿Cuál es la causa de la hipoxemia? A. Disminución del espacio muerto B. Aumento del shunt fisiológico C. Desequilibrio ventilación-perfusión D. Deterioro de la difusión 124. Varón de 45 años, sometido a un examen de retina, luego del cual presenta fotofobia y visión borrosa. Pupilas: midriasis bilateral, no reactivas a la luz ni a la acomodación. No otro trastorno oculomotor. ¿Cuál es el diagnóstico más probable? A. Midriasis farmacológica B. Parálisis del III par craneal C. Pupilas de Argyll Robertson D. Síndrome de Horner 125. Varón de 35 años, consulta por nauseas, cólico abdominal y diarreas posterior a la ingesta de leche y otros productos lácteos. Examen de heces: azúcares reductores ++. La causa más probable es A. Deficiencia de lactasa intestinal B. Deficiencia de amilasa pancreática C. Intolerancia a la proteína de la leche D. Deficiencia de la sacarasa-isomaltasa intestinal 126. Varón de 45 años, acude a consultorio por presentar desde hace un mes dolor tipo urente en epigastrio que aparece 5 a 6 horas después de la ingesta de alimentos, ocasionalmente el dolor lo despierta por las noches. Niega pérdida de peso y disfagia. Sin antecedentes relevantes. Al examen: no evidencia hallazgos significativos. ¿Cuál sería el diagnóstico más probable? A. Gastritis aguda B. Reflujo gastroesofágico C. Úlcera péptica D. Cáncer de estómago 127. Varón de 60 años, reside en Cerro de Pasco, consulta por cefalea, mareos y acúfenos desde hace 2 años, con empeoramiento en el último mes. Al examen: buena nutrición, hiperemia conjuntival, cianosis de dedos y labios. Laboratorio: Hb 22 g/dl Hto 66% y SaO2 88% ¿Cuál es el diagnóstico probable? A. Policitemia vera B. Mal de montaña crónico C. Hipoxemia refractaria D. Policitemia fisiológica secundaria 128. Escolar de 7 años con odinofagia y fiebre desde hace 3 días. Al examen: adenopatías cervicales dolorosas, orofaringe con tractos blanquecinos. ¿Cuál es el agente causal más frecuente? A. Adenovirus B. Streptococcus grupo B C. Streptococcus beta hemolítico grupo A D. Moraxella catarrhalis 129. Varón de 24 años, refiere contacto sexual casual sin protección hace 4 días, acude por presentar secreción mucopurulenta por la uretra asociada a disuria. ¿Cuál es el tratamiento empírico de elección? A. Penicilina G sódica + gentamicina B. Ceftriaxona + azitromicina C. Clindamicina + amikacina D. Penicilina benzatínica + clindamicina 130. Recién nacido de 36 semanas de edad gestacional y 12 horas de vida que inicia cuadro de dificultad respiratoria leve e hipotermia. Antecedente materno de RPM mayor de 18 horas. ¿Cuál es el diagnóstico más probable? A. Enfermedad de membrana hialina B. Taquipnea transitoria del recién nacido C. Hemorragia intraventricular D. Neumonía neonatal 131. Según el calendario de vacunación MINSA, ¿a que edad en años debe administrarse la dosis de refuerzo de la vacuna de varicela? A. 4 B. 10 C. 6 D. 5
  • 17. EXAMEN NACIONAL DE MEDICINA 2021 wwwww.qxmedic.com www.grupoqxmedic.com 132. Lactante de 9 meses con historia de diarreas frecuentes. Hace 3 días presenta vómitos y diarreas líquidas 4-6 veces al día. Al examen: peso 5 kg, pulso débil, hipoactivo, fascie de anciano, TCSC muy disminuido, llenado capilar > 3 segundos, ojos hundidos: AGA: pH 7.2, pCO2 25, HCO3 10 mEq/L. ¿Cuál es la condición asociada a su estado nutricional que empeora el pronóstico? A. Hiperkalemia B. Hipomagnesemia C. Hipocalcemia D. Hiponatremia 133. Varón de 32 años, con diagnóstico de Giardiasis intestinal, en el cuarto día de tratamiento con metronidazol 500 mg dos veces al día, luego de participar en una reunión social donde ingiere licor, lo llevan a la emergencia por presentar dificultad respiratoria, náuseas, vómitos, palpitaciones y excitación psicomotriz. ¿Cual es el diagnóstico más probable? A. Efecto antabus. B. Ingesta excesiva de alimentos grasos C. Excesiva dosis de metronidazol D. Toxicidad por alcohol 134. Lactante de 5 meses con pobre ganancia ponderal. Polipneico y diaforético durante la lactancia. Al examen: taquicárdico, soplo sistólico II/IV en mesocardio, borde hepático a 3 cm debajo del reborde costal. ¿Cuál es el diagnóstico probable? A. Insuficiencia cardiaca B. Ducto arterioso persistente C. Miocarditis viral D. Enfermedad de Kawasaki 135. Pretérmino de 35 semanas de edad gestacional y 2 horas de vida. Peso al nacer 2000 g. Presenta irritabilidad, temblor, dificultad en la succión y 30 minutos después se torna letárgico. Hemoglucotest: 30 mg/dl. Se decide colocar dextrosa al ……% 200 mg/kg, seguido por una infusión de 6-8 mg/kg/min. A. 50 B. 10 C. 33 D. 5 136. Varón de 58 años, acude por haber presentado convulsión tónico clónica generalizada con relajación de esfínteres que duró aproximadamente 30 segundos. Al examen se encuentra algo confuso y no recuerda bien lo sucedido. No tiene antecedentes de anteriores convulsiones. ¿Cuál es el examen auxiliar preferencial para confirmar el diagnóstico? A. Resonancia magnética de cerebro B. Electroencefalograma C. Punción lumbar D. Tomografía computarizada de cerebro 137. Chef de 30 años sufre quemaduras de 2do y 3er grado en cara y cuello. Al examen: edema de mucosa oral, esputo de color negro, restos carbonizados en boca y fosas nasales, estridor laríngeo y cianosis. ¿Cuál es la acción prioritaria? A. Intubación endotraqueal B. Radiografia de tórax C. Tomografía cervical D. Corticoides EV 138. Gestante de 28 semanas acude con dolor tipo contracción persistente y ginecorragia. Refiere que no siente movimientos de su bebé, G2 P1001, Cesárea hace 10 meses por causa similar ala gestación actual. Al examen: PA 130/90 mmHg, FC 100 x¨, FR 16 X', LCF 102 x. ¿Cuál sería el factor principal que condiciona la complicación del actual embarazo? A. Periodo intergenésico corto B. Trastorno hipertensivo del embarazo C. Placenta previa D. Cesárea anterior 139. Varón de 45 años, estando en Emergencia presenta súbita pérdida de conciencia con ausencia de pulso y apnea. En el monitor cardíaco se evidencia ritmo acelerado (ver imagen). ¿Cuál es la mejor opción terapéutica? A. Desfibrilación B. Cardioversión C. Ablación D. Betabloqueante EV
  • 18. EXAMEN NACIONAL DE MEDICINA 2021 wwwww.qxmedic.com www.grupoqxmedic.com 140. Varón de 60 años con antecedente de colitis ulcerosa, desde hace una semana presenta diarreas, indicándose con loperamida sin éxito. Al examen: FC 120 x’, T 39 °C. soporoso, distensión abdominal con RHA ausentes. Hemograma Leucocitosis con neutrofilia Hb 7 gr/dl. ¿Cuál es el diagnóstico más probable? A. Estenosis sigmoidea B. Isquemia mesentérica C. Diverticulitis aguda D. Megacolon tóxico 141. Neonato del 14 días acude a control de niño sano. Al examen buen estado general sin fiebre, cordón umbilical no maloliente, sin secreciones y con restos de nitrato de plata. Resto del examen sin alteraciones. Antecedentes no relevantes. Pruebas metabólicas normales. ¿Cuál es el diagnóstico probable? A. Problemas de inmunidad humoral B. Retraso secundario a problemas del uraco C. Retraso de caída del cordón sin patología asociada D. Retraso por defecto congénito de la adhesividad leucocitaria 142. Mujer 35 años, con artritis de rodilla izquierda el líquido sinovial con tinción de Gram, demuestran bacterias grampositivas agrupadas en racimos de uva. ¿Cuál es la mejor conducta a seguir? A. Drenaje y antibióticos vía parenteral B. Antibióticos vía parenteral con antinflamatorios C. Drenaje y antinflamatorios parenterales D. Lavado intra articular continuo 143. Gestante de 24 semanas, asintomática, con prueba de tuberculina positiva y RX de tórax normal. Esposo acaba de ser diagnosticado de TBC, BK Positivo. ¿Cuál es la conducta a seguir? A. Radiografía en seis meses B. TEM de tórax en un mes C. Isoniacida profiláctica D. BK seriado por tres tomas 144. Mujer de 25 años con episodios agudos de migraña a razón de más de 4 episodios por mes. ¿Cuál sería la mejor medida terapéutica para evitar la recurrencia de los episodios? A. Tramadol B. Sumatriptan C. Propanolol D. Ibuprofeno 145. Segundigesta de 14 semanas con sobrepeso, acude a su primer control prenatal (CPN). Su primer hijo nació con 4400 g de peso. Se le realiza el test de detección de diabetes con 50 g de azúcar, saliendo negativo. ¿Cuál es la conducta a seguir? A. Repetir la prueba entre las 24 y 28 semanas B. Solicitar un test de tolerancia a la glucosa C. Medir glucosa cada tres semanas D. Realizar CPN normal 146. Pre-escolar de 3 años con familia en extrema pobreza hace 3 meses presenta fatiga, anorexia, náuseas, dolor abdominal, diarreas y calambres. Recibe tratamiento variados sin mejoría. Antecedentes: no recibió lactancia materna. Dieta pobre en carnes. Al examen: muy adelgazado, edematoso, pálido y presenta marcada disminución de los reflejos tendinosos profundos. ¿Qué déficit vitamínico es el más probable? A. Riboflavina B. Tiamina C. Piridoxina D. Niacina 147. Varón de 38 años, con IMC de 38, obeso desde los 17 años. Ha probado múltiples tratamientos sin éxito, pierde peso y posteriormente lo recupera. comienza a automedicarse con Orlistat (inhibidor de absorción de grasa), consulta sobre qué otro tratamiento podría adicionar. Su recomendación sería: A. Dieta vegana B. Metformina C. Dieta mediterránea D. Ejercicios aeróbicos
  • 19. EXAMEN NACIONAL DE MEDICINA 2021 wwwww.qxmedic.com www.grupoqxmedic.com 148. Niño de 6 años, llevado por su madre por referir dolor en oído desde hace 3 horas. Otoscopía: insecto vivo que se mueve en el canal auditivo externo. ¿Cuál es el manejo inicial? A. Lavado de oído con solución salina B. Extracción instrumental con pinza C. Instilación ótica de aceite mineral D. Irrigación ótica con agua oxigenada 149. Primigesta de 40 semanas en pródromos de labor, acude por disminución de movimientos fetales. Se realiza NST no reactivo y variabilidad disminuida. Perfil biofísico (PBF): normal ¿Cuál es el siguiente paso a seguir? A. Cesárea de emergencia B. Test estresante C. Repetir NST en 6 horas D. Repetir PBF en 72 horas 150. Durante su rotación de ginecología y obstetricia, usted observó un aumento significativo de casos de endometritis puerperal. ¿Qué medida ayudaría a prevenir estas infecciones? A. Usar sonda Foley durante el parto B. Fenobarbital C. Disminuir los tactos vaginales innecesarios D. Rasurar el vello púbico a todas las gestantes 151. Recién nacido a término nacido por cesárea debido a bradicardia fetal. A las 2 horas de vida presenta movimientos tónico clónicos generalizados. ¿Cuál es el medicamento de elección? A. Levetiracetam B. Fenobarbital C. Difenilhidantoina D. Midazolam 152. Varón de 20 años sufre quemadura con agua caliente en el antebrazo derecho. Al examen: lesiones eritematosas, ampulares y dolorosas que palidecen a la presión. ¿Qué medida terapéutica está indicada inicialmente? A. Antibióticos sistémicos B. Aplicar antibióticos tópicos C. Vacuna antitetánica D. Reposición hídrica según fórmula de Parkland 153. Varón de 17 años con antecedente de rinitis alérgica, consulta por dermatitis atópica ¿Cuál es la terapia tópica recomendada? A. Higiene constante (≥3 veces por día) B. Aplicación de emolientes e hidratantes C. Corticoides tópicos de alta potencia D. Neomicina en ungüento 154. Enfermero que labora en UCI, desde hace 5 días presenta fiebre, tos, cefalea, mialgia y odinofagia asociadas a debilidad general. Al examen: FR 22 x', SaO2 91% ¿Qué característica biológica corresponde al agente causal de esta enfermedad? A. Se une al receptor ACE 2 a través de la proteína S B. Su material genético ingresa a la célula como DNA C. Su replicación es controlada por el DNA del neumocito D. La proteína M le da su morfología típica 155. Mujer del 32 años, sufre contusión grave toraco abdominal. Al examen: equimosis en tórax anterior bajo y epigastrio, palidez intensa de piel y mucosas, compromiso de conciencia, ingurgitación yugular marcada. PA 60/30 mmHg, pulso imperceptible, latidos cardiacos poco audibles SaO2 89% Abdomen no distendido, difícil evaluar por estado de conciencia. EKG: complejos QRS de bajo voltaje en todas las derivaciones. Se intenta pericardiocentesis sin éxito. ¿Cuál es el procedimiento prioritario a realizar? A. Toracotomía exploratoria B. Toracoscopía diagnóstica C. Ventana pericárdica D. Decorticación pericárdica 156. Paciente mujer de 35 años de edad que trabaja en ventas, desde hace un año presenta 3 a 4 veces al mes episodios de dolor cólico abdominal y borborigmos que ceden con la defecación, acompañado de distensión abdominal ocasional y alternancias de diarrea y estreñimiento. El examen físico es normal. ¿Cuál es su impresión diagnóstica? A. Colon irritable B. Distensión abdominal funcional C. Dispepsia funcional D. Colecistitis crónica
  • 20. EXAMEN NACIONAL DE MEDICINA 2021 wwwww.qxmedic.com www.grupoqxmedic.com 157. En un colegio secundario el 80% de alumnos del último año presentan sobrepeso u obesidad. Usted es el médico responsable de la estrategia sanitaria de enfermedades no transmisibles del establecimiento de salud. ¿Qué tipo de intervención inmediata priorizará? A. Programación de consultas nutricionales B. Comunicación educativa grupal C. Clausura del quiosco escolar D. Difusión radial de factores protectores 158. Mujer de 48 años, que hace 5 días presenta dolor en epigastrio luego de la ingesta de comidas grasas, 24 horas después aparece coloración amarillenta en piel y escleras. Colangioresonancia: múltiples cálculos en la vía biliar principal, uno grande de 1.3 cm de diámetro impactado en colédoco distal. ¿Cuál es la conducta terapéutica a seguir? A. Papilotomía endoscópica B. Colecistectomía C. Exploración de vía biliar y drenaje Kherr D. Colecistostomía 159. Recién nacido, en su evaluación urogenital se encuentra pene curvo con fimosis, orificio uretral que desemboca en región ventral del cuerpo del pene ¿Qué se esperaría encontrar en el examen escrotal? A. Varicocele B. Bolsa escrotal vacía C. Agenesia de epidídimo D. Testículos normales 160. Varón de 45 años con colitis ulcerativa de 3 años de evolución. Hospitalizado por exacerbación de la colitis, consunción, dolor abdominal y apatía. Perfil tiroideo, T3 total y libre disminuido, T4 libre y TSH normal. ¿Cuál es el diagnóstico más probable? A. Hipotiroidismo primario B. Sindrome del eutiroideo enfermo C. Hipotiroidismo subclínico D. Hipertiroidismo subclínico 161. Mujer de 60 años, obesa, refiere prurito vulvar y escaso flujo vaginal. Al examen: vulva congestiva, eritematosa, con sequedad de sus paredes. ¿Cuál es el diagnóstico más probable? A. Vulvovaginitis inespecífica B. Vulvovaginitis candidiásica C. Vaginosis bacteriana D. Vaginitis atrófica 162. Paciente de 25 años asintomática. En una ecografía de rutina presenta en ovario derecho masa quística unilocular de 4 cm sin tabiques ni excrecencias. ¿Cuál es la conducta a seguir? A. Quistectomía B. Observación C. Solicitar alfa feto proteína D. Prescribir anticonceptivos orales 163. Varón de 38 años, refiere intenso dolor ocular, lagrimeo y enrojecimiento mientras trabajaba con una amoladora eléctrica sin contar con protector facial. Al examen: cuerpo extraño metálico que atraviesa córnea, iris y cristalino. ¿Cuál es el manejo inicial? A. Colirio analgésico B. Analgesia endovenosa C. Parche compresivo D. Tonometría 164. Primigesta de 33 semanas, consulta por cefalea de un día de evolución. Al examen: PA 170/110 mm Hg, Hb 10.5 g/dl, plaquetas 95000/mm3, TGO 400, TGP 300 y LDH 900. Esquistocitos en el frotis de sangre periférica. Recibe nifedipino vía oral sin respuesta. ¿Cuál es la conducta más adecuada? A. Completar la maduración pulmonar fetal B. Terapia con antihipertensivos y sulfato de magnesio C. Manejo expectante con controles cada 48 h D. Finalización inmediata de la gestación
  • 21. EXAMEN NACIONAL DE MEDICINA 2021 wwwww.qxmedic.com www.grupoqxmedic.com 165. Mujer de 45 años, desde hace 10 días presenta disfagia para alimentos sólidos. Desde hace 48 horas fiebre de 39 °C, trismus y dificultad respiratoria. Al examen: desviación medial del paladar blando. Hemograma con 18000 leucocitos y 7% abastonados. TAC cervical: colección de 30 ml en orofaringe. ¿Cuál es el diagnóstico más probable? A. Absceso dentario B. Absceso periamigdaliano C. Angina de Ludwig D. Amigdalitis aguda 166. Varón de 45 años, víctima de accidente de tránsito es llevado a la emergencia. Al examen: lúcido, muy quejumbroso, equimosis a nivel de la novena y décima costilla izquierda con crepitación y dolor a la palpación. ECO FAST: Líquido libre en cavidad aproximadamente 300 ml. ¿Cuál es el órgano que está probablemente lesionado? A. Cola de páncreas B. Fondo gástrico C. Ángulo esplénico del colon D. Bazo 167. Mujer de 30 años se encuentra amamantando a su bebe. ¿Qué respuesta hormonal se espera en la madre? A. Aumento de la secreción de oxitocina B. Disminución de secreción de FSH C. Disminución de la secreción de prolactina D. Aumento de la secreción de vasopresina 168. Gestante de 37 semanas en tratamiento con TARGA, con carga viral de 1500 copias/ml, acude a Emergencia con dilatación de 1 cm, altura de presentación -1, borramiento al 100% y membranas ovulares íntegras. ¿Cuál es la conducta más riesgosa para el recién nacido? A. Parto por cesárea B. Zidovudina endovenosa C. Parto por vía vaginal D. Continuar con TARGA 169. Mujer de 36 años acude a emergencia por presentar desde hace 6 días, escalofríos, sensación de alza térmica y malestar general, disuria y tenesmo. Examen Físico: T 38.5° C, dolor difuso en abdomen y pelvis. Examen de Orina: Presencia de nitritos y cilindros leucocitarios. ¿Cuál es el diagnóstico probable? A. Pielonefritis B. Cistitis C. Vaginitis D. Uretritis 170. Niña de 4 años presenta desde hace un mes palidez, fatiga y dolor óseo en piernas. Al examen: hepatoesplenomegalia. Laboratorio: Hb 7 g/dl, plaquetas 20000 mm3, leucocitos 35000 mm3, linfoblastos 1%. ¿Cuál es la conducta a seguir? A. Biopsia hepática B. Radiografía de extremidades C. Ecografía abdominal D. Aspirado de médula ósea 171. Mujer de 28 años acude por emergencia con herida profunda en muslo izquierdo de 4 días de evolución. Dolor intenso en muslo izquierdo y alza térmica, siendo atendida en posta médica. Examen: herida cruenta con flogosis, vesículas de color marrón oscuro, secreción de agua con aspecto de lavado de carne. ¿Cuál es el diagnóstico más probable? A. Seroma aplastronado B. Absceso multilocular C. Fasceitis necrotizante D. Hematoma infectado 172. Puérpera inmediata de 27 años, sometida a cesárea de emergencia presentando sangrado de 500 ml en el acto quirúrgico. Tres horas después hipotensión arterial de 60/40 mmHg, FC 120 x' e incremento de la palidez. El tipo de shock que estaría presentando es de A. Continente (distributivo) B. Obstructivo C. Bomba (cardiogénico) D. Contenido (hipovolémico)
  • 22. EXAMEN NACIONAL DE MEDICINA 2021 wwwww.qxmedic.com www.grupoqxmedic.com 173. Mujer de 60 años sin antecedentes de enfermedad vascular previa, refiere dolor en pie y región sural derecha. Al examen: ausencia de pulso pedio y poplíteo con disminución de la sensibilidad, frialdad en pie e incapacidad para mover el grupo muscular afectado. ¿Cuál es el diagnóstico probable? A. Embolia arterial B. Insuficiencia venosa profunda C. Trombosis ateroesclerótica D. Espasmo arterial 174. Mujer de 30 años, desde hace 5 meses tiene molestias urinarias recurrentes. Examen de orina muestra pH ácido, leucocitos 60 x campo. Urocultivo negativo ¿Cuál es la sospecha diagnostica? A. Absceso perirrenal B. Cistitis crónica C. Pielonefritis crónica D. Tuberculosis urogenital 175. Mujer de 25 años sexualmente activa con amenorrea de 9 semanas, desde hace 24 horas presenta dolor tipo cólico en hipogastrio y sangrado escaso. Al examen: funciones vitales estables, ansiosa con facies dolorosa, cérvix blando, cerrado, de aspecto violáceo con escaso sangrado ¿Cuál es el procedimiento a realizar? A. Indicar culdocentesis B. Solicitar ecografía transvaginal C. Solicitar B-hCG D. Preparar para legrado uterino 176. Varón de 30 años hace 12 horas sufrió golpe en la cabeza con pérdida de conocimiento, al momento presenta cefalea global de intensidad moderada, vómitos explosivos, mareos y fotofobia. Fondo de ojo: edema de papilas. Glasgow 10 y descendiendo. ¿Cuál indicación es prioritaria? A. Solución cristaloide isosmótica B. Dimehidrinato sódico C. Antiinflamatorios no esteroideos D. Solución salina hipertónica 177. Lactante de 1 mes hospitalizado por presentar paroxismo de tos con estridor y ahogo desde hace una semana. Antecedente: hermano mayor presentó episodio similar hace dos semanas. ¿Cuál es el tratamiento antibiótico de primera línea? A. Cotrimoxazol B. Amoxicilina C. Azitromicina D. Clindamicina 178. Varón de 76 años que, desde hace 1 año, presenta polaquiuria, disuria y urgencia vesical, hace dos meses vaciado incompleto de vejiga y goteo postmiccional. Al tacto: próstata aumentada de tamaño en forma difusa. PSA 5 ng/ml. ¿Cuál es el tratamiento inicial? A. Agonistas de la GnRH B. Resección transuretral de la próstata C. Bloqueadores alfa adrenérgicos D. Vaporización de la próstata con láser 179. Varón de 60 años, procedente de zona rural de Puno, con un día de dolor abdominal intenso y marcada distensión, asociados a vómitos fecaloideos. RX de abdomen: signo de grano de café. No antecedentes quirúrgicos. ¿Cuál es el diagnóstico más probable? A. Intususcepción intestinal B. Perforación de víscera hueca C. Diverticulitis aguda D. Vólvulo de sigmoides 180. Varón de 67 años trabaja 15 años como obrero en una fábrica de papel. Presenta disnea y tos desde hace 1 año. Examen: PA 130/80 mmHg, FR 26 x' acropaquias y crepitantes gruesos en bases de ambos hemitórax. RX tórax: imágenes reticulonodulares basales y bilaterales, pulmones pequeños, SaO2 94% en reposo y 72% al esfuerzo. ¿Cuál es el diagnóstico más probable? A. Enfisema pulmonar B. Enfermedad pulmonar intersticial difusa C. Enfermedad pulmonar obstructiva crónica D. Hipertensión pulmonar